Vous êtes sur la page 1sur 54

SET A Seat No.

____
---------------------------------------------------------------------------------------------------------------------------------
NURSING PRACTICE 1:
FOUNDATION OF PROFESSIONAL NURSING PRACTICE
Direction: Choose the letter of the BEST answer by shading the corresponding letter of your choice on the answer sheet
provided. STRICTLY NO ERASURE!

Situation 1: Mr. Martin, 71 years old was suddenly rushed to the hospital because of severe chest pain. On admission,
he was diagnosed to have acute myocardial infarction and was placed in the ICU.

1. While in the ICU, he executes the document tat list the medical treatment he chooses to refuse in case his condition
becomes severe to a point that he will be unable to make decisions for himself. This document is:
A. living will C. last will and testament
B. informed consent D. power of attorney

2. After one day, the patient’s condition worsened and feeling hopeless. He requested the nurse to remove the oxygen.
The nurse should:
A. follow the patient because it is his right to die gracefully
B. follow the patient as it is his right to determine the medical regimen he needs
C. refuse the patient and encourage him to verbalize hid feelings
D. refuse the patient since euthanasia is not accepted in the Philippines

3. Euthanasia is an ethical dilemma which confronts nurses in the ICU because:


A. the choices involved do not appear to be clearly right or wrong
B. a clients legal right co-exist with the nurse’s professional obligation
C. decisions has to be made based on societal norms.
D. decisions has to be mad quickly, often under stressful conditions

4. A nurse who supports a patient and family’s need to make decisions that is right for them is practicing which of the
following ethical principles?
A. Autonomy B. confidentiality C. privacy d. truthfulness

5. Mr. Martin felt better after 5 days but recognizing the severity of his illness, he executes a document authorizing the
wife to transact any form of business in his behalf in addition to all decisions relative to his confinement his document is
referred to as:
A. power of attorney C. informed consent
B. living will D. medical records

Situation 2: Miss Castro was recently appointed chief nurse of a 50-bed government hospital in Valenzuela. On her first
day of duty, she tried to remember the elements of administration she learned from her basic nursing education.

6. One of the first things Ms. Castro did was to engage her until in objective writing, formulating goals and philosophy of
nursing service. Which activities are MOST appropriately described to which elements of administration?
A. planning B. controlling C. directing d. organizing

7. In recognizing the Department of Nursing, she comes up with the organizational structure defining the role and
function of the different nursing positions and line-up the position with qualified people. This is included in which element
of administration:
A. monitoring B. evaluation C. organizing d. planning

8. After one month, she and her management committee assess the regulatory measures taken and correct whatever
discrepancies are found. This is part of which element of administration:
A. monitoring B. organizing C. evaluation d. planning

9. Revaluation and administrative process is BEST described as:

1
A. a continuing process of seeing that performance meets goals and targets
B. obtaining commitment of members to do better
C. informing personnel how well and how much improvement has been made
D. follow-up of activities that have been studied

10. In all of the various administrative functions, which of the following management skill is demanded efficiently and
effectively of Ms. Castro?
1. Decision making skills 2. Forecasting skills
3. Auditing skills 4. Communications skills

A. 2 & 3 B. 1 & 4 C. 1 & 2 D. 2 & 4

Situation 3: Meldy. 40 years old. is waiting for her doctor’s appointment at the clinic where you work.

11. You are to interview her as an initial nursing action so that you can.
A. Document important data in her client records for health team to read.
B. Gather data about her lifestyle, health needs, lifestyle, health needs and problems to develop
plan of care
C. provide solutions to her immediate health concern
D. identify the most appropriate nurse diagnosis for her heath problem

12. During the interview, Meldy experiences a sharp abdominal pain on the right side of her abdomen. She further tells
you that an hour ago, she ate fatty food and this had happened many times before. You will record this as:
A. Client complains of intermittent abdominal pain an hour alter eating fatty foods
B. After eating fatty food the client experienced severe abdominal pain
C. Client claims to have sharp abdominal pains after eating fatty food unrelieved by pain medication
D. Client reported sharp abdominal pain on the right upper quadrant of abdomen an hour after
ingestion of fatty foods.

13. Meldy tells you that she has been on a high protein / high fat / low carbohydrate diet order to lose weight and that
she has successfully lost 8 lbs during the past two weeks. In planning a healthy balanced diet for her, you will:
A. Encourage her to eat well-balanced diet with a variety of food from the major food groups and
take plenty of fluids.
B. Ask her to shift to a macrobiotic diet rich in complex carbohydrates.
C. Encourage her to cleanse her body toxins by changing a vegetarian diet with regular exercise.
D. Encourage her to eat a high carbohydrate, low protein diet and low fat diet.

14. You learn that Meldy drinks 5-8 cups o coffee a day plus cola drinks. Because she is in her pre-menopausal years, the
nurse instructs her to decrease consumption of coffee and cola preparation because:
A. these products increase calcium loss from the bones
B. These products have stimulant effect n the body
C. these products encourage increase in sugar consumption
D. these products are addicting

15. Health education plan for Meldy stresses prevention of NCD or Non-communicable diseases that are influenced by
lifestyle. These include the following EXCEPT:
A. Cancer B. DM C. Osteoporosis D. Cardiovascular diseases

Situation 4: Changes in technology, the nation’s economy and the increasing number of population have brought about
changes in the Health Care System.

16. At present, government hospitals are expected to offer comprehensive health services to include illness prevention
and health promotion. In which of the following unit of services are these services integrated?
A. Wellness center C. Rehabilitation Center
C. Intensive Care unit D. newborn screening unit

17. Which of the following is the MOST recent government initiative to help subsidize the cost of health services for both
the employed and the unemployed?
2
A. National Health Insurance Act C. Medicare Act
B. Worker’s Compensation Act D. Magna Carta for Public Health Workers

18. The top ten morbidity cases in the Phil. Include TB, diarrhea among children to name a few. Many of these conditions
are preventable and have implications are preventable and have implications in the development of which nursing
competencies?
A. Execution of nsg. procedure and technique
B. Therapeutic use of self
C. Administration of treatment and medication
D. Health education

19. The cost of hospitalization is getting more expensive and unaffordable to many of our people. These facts will MOST
LIKELY bring about development in which of the following?
A. acute services C. home care services
B. managed care services D. advance practice nursing

20. Which of the following latest trend has expanded health services based on prepaid fees with emphasis on health
promotion and illness prevention?
A. Government Insurance Plan C. Health Maintenance Organization
B. Preferred Provider Organization D. Private Insurance Plan

Situation 5: It is Safety Awareness Week in the Community and the nurse checks on the presence of hazards at home.
The nurse plan is to have the residents themselves identify the physical hazards in their own homes.

21. Which of the following is NOT a physical hazard in the home?


A. unstable and slippery stairway
B. large windows that allow good ventilation
C. obstacle people cam trip over like door mats, rugs, electric cords
D. inadequate lighting in and out of the house

22. Risk factors exist for each of the different developmental levels. From infancy to preschool age, the most common
cause of death is injury rather than disease. To protect children from harm, that parents should be aware that MOST
injuries for this age group are due to:
A. Accidents at home caused by the swallowed poisonous materials, small objects, exploring
electrical sockets
B. Accidents from self inflicted wounds
C. accidents from sports related activities at school or the neighborhood
D. accidents in the Playground Park, school and presence of strangers who may abduct of molest the child.

23. To promote safety at home, the nurse identifies ways and means of “child proofing” the house. Which of the following
is NOT safe?
A. apply child proof caps and medicine bottles and chemicals
B. covering electrical outlets, tying up long and loose electrical and telephone cords, securing cabinets
or doors within reach o the child
C. giving colorful grocery bags to play with or to store toys and materials
D. removing objects that the child could easily dismantle and swallow like small parts of a mechanical
toy, buttons, materials inside, stuffed animals, liquid chemicals.

24. The nurse knows that a person’s hygienic practices are influenced family customs and traditions. Which of the
following is NOT part of Basic Hygienic Practices?
A. bathing practices, frequency and time, care of eyes, ear and nose
B. oral hygiene practices such as brushing and flossing teeth, gum care
C. care of skin with lesions, cuts with infection
D. hair and skin such as washing hair and face, feet, hand and nail care

25. Falls are the common home accident among elderly and these are due to physical limitations imposed by aging and
some hazards in the home setting. The nurse reduces the risk of falling through the following EXCEPT:
A. rearranging furniture frequently
3
B. having the bed or mattress close to the floor
C. providing a nonskid and well fitted shoes or slippers
D. having a call bell within the persons reach and answering call bells immediately

Situation 6: Eileen, 45 years old is admitted to the hospital with a diagnosis of renal calculi. She is experiencing severe
flank pain, nauseated and with a temperature of 39 0C.

26. Given the above assessment data, the most immediate goal of the nurse would be which of the following?
A. Prevent urinary complication C. Alleviate pain
B. maintains fluid and electrolytes D. Alleviating nausea

27. After IVP a renal stone was confirmed, a left nephrectomy was done. Her post operative order includes “daily urine
specimen to be sent to the laboratory” . Eileen has a foley catheter attached to a urinary drainage system. How will
you collect the urine specimen?
A. remove urine from drainage tube with sterile needle and syringe and empty urine from the
syringe into the specimen container
B. empty a sample urine from the collecting bag into the specimen container
C. disconnect the drainage tube from the indwelling catheter and allow urine to flow from catheter into
the specimen container.
D. disconnect the drainage the from the collecting bag and allow the urine to flow from the catheter into
the specimen container.

28. Where would the nurse tape Eileen’s indwelling catheter in order to reduce urethral irritation?
A. to the patient’s inner thigh C. to the patient’
B. to the patient’s lower thigh D. to the patient lower abdomen

29. Which of the following menu is appropriate for one with low sodium diet?
A. instant noodles, fresh fruits and ice tea
B. ham and cheese sandwich, fresh fruits and vegetables
C. white chicken sandwich, vegetable salad and tea
D. canned soup, potato salad, and diet soda

30. Howe will you prevent ascending infection to Eileen who has an indwelling catheter?
A. see to it that the drainage tubing touches the level of the urine
B. change he catheter every eight hours
C. see to it that the drainage tubing does not touch the level of the urine
D. clean catheter may be used since urethral meatus is not a sterile area

Situation 7: Miss Tingson is assigned to Mang Carlos, a 60 year old newly diagnosed diabetic patient. She is beginning
to write objectives of her teaching plan.

31. Which of the following objectives is written in behavioral terms?


A. Mang Carlos will know about diabetes related to foot care and the techniques and equipments necessary to
carry it out
B. Mang Carlos daughter should learn about DM within the week
C. Mang Carlos wife needs to understand the side effects of insulin
D. Mang Carlos sister will be able to determine in two days his insulin requirement based on
blood glucose levels obtained from glucometer

32. Which of the following is the BEST rationale for written objectives?
A. ensure communication among staff members
B. facilitate evaluation of the nurse’s performance
C. ensure learning on the part of the nurse
D. document the quality of care

33. Which of the following behavior BEST contribute to the learning of Mang Carlos regarding his disease condition?
A. frequent use of technical terms for familiarization
B. drawing him into discussion about diabetes
4
C. detailed lengthy explanation about his condition
D. loosely structured teaching session

34. Miss Tingson should encourage exercise in the management of diabetes, because it:
A. decrease total triglyceride levels C. lowers blood glucose
B. improves insulin utilization D. accomplishes all of the above

35. The chief life-threatening hazard for surgical patient with uncontrolled diabetes is:
A. dehydration B. hypertension C. hypoglycemia D. glucosuria

Situation 8: Caring for the perioperative patient.

36. An appendectomy during a hysterectomy would be classified as:


A. Major, emergency, diagnosis C. Minor, elective, ablative
B. major, urgent, palliative D. minor, urgent, reconstructive

37. An informed consent is required for:


A. closed reduction of a fracture C. irrigation of the external ear canal
B. insertion of intravenous catheter D. urethral catheterization

38. The circulating nurse’s responsibilities, in contrast to the scrub nurse’s responsibilities, include:
A. assisting the surgeon C. setting up the sterile tables
B. monitoring aseptic practices D. all of the above functions

39. The primary nursing goal in the immediate postoperative period is maintenance of pulmonary function and prevention
of:
A. Laryngospasm C. hypoxemia and hypercapnea
B. hyperventilation D. pulmonary edema ad embolism

40. Unless contraindicated, any unconscious patient should be positioned:


A. flat on his of her back, without elevation of the head, to facilitate frequent turning and minimize pulmonary
complications
B. in semi-fowlers position, to promote respiratory function and reduce the incidence of orthostatic hypotension
when the patient can eventually stand.
c. in fowler’s position, which most closely stimulates a sitting position, thus facilitating reparatory as well as
gastrointestinal functioning.
D. on his or her side without a pillow at the patient’s back and his or her chin extended to minimize
the danger of aspirations

Situation 9: To prevent further injury to patients with problems of immobility / mobility, the nurse should observe
certain principles of body mechanics for herself and her patients.

41. Which of the following are appropriate goals for client with positioning and mobility needs?
A. developing of contractures C. sensory alterations
B. proper body alignment D. decrease in activity tolerance

42. Which for the following would MOST likely cause injury to the nurse when moving the patient from bed to the
wheelchair?
A. bending at the knees C. using body weight to assist with the movement
B. standing with feet together D. standing with feet apart

43. Which of the following is the CORRECT guideline when positioning patients?
A. put pillows above a joint to immobilize it
B. position of the joint should be slightly extended
C. joints of patient to be supported with pillow
D. patient’s position should be changed at least three or four times a day

5
44. Which of the following can be used by clients with problem of immobility to enable them raise their body from bed to
wheelchair or perform some bed exercises?
A. sandbag B. side-rail C. trochanter roll D. trapeze bar

45. The importance of forcing fluids with an immobilized patient is to:


A. prevent pneumonia C. prevent skin breakdown
B. prevent urinary stasis D. maintain peristalsis

46. Which of the following is the least nursing activity in performing assessment of the patient?
A. laboratory test C. Health history
B. physical examination D. systemic review

47. One of the responsibilities of Mr. Lata, RN, an industrial nurse, is to conduct physical head-to-toe assessment of a
newly hired factory worker. As part of the assessment, he took the vital signs. Which of the four assessment techniques
did he utilize?
1. auscultation 3. palpation
2. percussion 4. inspection
A. 1,2 & 3 B. 3 &4 C. 1 & 2 D. 1,3 & 4

48. Which of the following are the purpose of performing a physical assessment?
1. gather baseline data about the client’s health
2. confirm and identify nursing diagnosis
3. evaluate physiological outcome of care
4. make clinical judgment of patients’ diagnosis
A. 1,2 & 4 B. 2, 3 & 4 C. 1, 3 & 4 D. 1,2 & 3

49. Which of the following should be given the HIGHEST PRIORITY before physical examination is done to a patient?
A. preparation of the equipment C. preparation of the environment
B. psychological preparation of the client D. physical preparation of the client

50. During the assessment phase of the nursing process, the nurse is concerned with:
A. interpreting data
B. designing nursing strategies
C. establishing a data base
D. comparing client responses with the anticipated outcome

Situation 11: The nurse is responsible to accurately records and reports patient’s progress. She is able to communicate
to the other member of the team by documenting the nursing care plan and the appropriate nursing intervention.

51. A main function of the patient’s records is to:


A. prepare the nurse for the shift worked
B. serve as a record of financial charges
C. serve as a vehicle for communication
D. ensure that the message is received

52. When the nurse writes in the chart and discovers an error has been made, which is the BEST approach?
A. erase the erroneous material
B. carefully ink out the erroneous material
C. place as asterisk next to the statement, then footnote it
D. draw a straight line through the error and initial it.

53. Which of the following persons cannot have the access to the patient record?
A. physical therapist C. the patient
B. lawyer of the family D. speech therapist

54. POMR charting is different from traditional method because of which of the following practices?
1. SOAP charting 3. narrative charting
2. use of flow sheet 4 . use of checklist
6
A. 3 & 4 B. 1 & 2 C. 1 & 3 D. 2 & 3

55. Which of the following qualities are relevant in documenting patients care?
1. Accuracy and consciousness
2. thoroughness and currentness
3. systematic and orderly
4. legibly, properly dated and signed
5. use of locally accepted abbreviation
A. 1,3,4 & 5 B. 2,3,4 & 5 C. 1,2,3 & 5 D. 1,2,3 & 4

Situation 12: The practice of primary nurse in primary nursing is preferred by many nurses because it supports
professional autonomy and accountability of the nurses>

56. What is the function of the primary nurse in primary nursing?


A. acts as patient advocate and coordinate the health care team for specific group of patients
B. act as the charge nurse, organizing staff assignments and help in solving problem in the unit
C. plans and coordinate the patient care assigned to her from admission to discharge
D. coordinates the care given to a group of patients by support staff

57. Primary nursing is MOST advantageous and satisfying to the patient and nurse because of which of the following
principles?
A. autonomy and authority for planning care are best delegated to a nurse
B. accountability is clearest since our nurse is responsible for the overall plan and implementation of care
C. the holistic approach provides fro a therapeutic relationship continuity of care and efficient
nursing care
D. continuity of patients care promotes efficient nursing care.

58. Which is the role of the associate nurse in primary nursing?


A. over-all manager of the unit
B. responsible for the over-all care of the patient during off days of primary nurse
C. patient advocate in the health care team
D. coordinator of comprehensive, holistic patient care

59. In primary nursing, the nurse is responsible for which of the following group of patient?
A. the whole ward
B. small group of patient like 3-5 patients
C. big group of patients like 10-15 patients
D. the whole unit

60. In primary nursing who among the following is needed to her leadership and quality control in the ward?
A. the chief nurse C. the nurse supervisor
B. the head nurse D. the service director

Situation 13: Your nursing unit plans to conduct a study on the use of structured preoperative preparation in lessening
the demand for post-operative pain medications.

61. Which of the following research activities should you initially do?
A. find out from interview how many patients are willing to participate
B. get the permission from the hospital director
C. review literature on the topic
D. prepare the tool for collecting data

62. Which of the following statements do NOT contribute to the researchabilty of your proposed problem?
A. potential use of findings C. well-defined problem statement
B. readability of findings D. measurability of variables

63. A study /research table should NOT contain which of the following ?
A. categories of data collected C. specific title of table
7
B. relevant rows and columns D. names and sample of the selected

64. Which of the following actions will facilitate analysis of research data?
A. consult a physician C. consult a complete expert
B. consult an adviser D. categorize data collected

65. The research methodology that is appropriate for the above problem would be:
A. descriptive B. normative C. experimental D. quasi experimental

Situation 14: The nurse meets a new client, Mr. Principe, 50 years old. During the initial interview, the nurse begins to
feel irritated towards the client. Shortly after, he becomes uncomfortable and politely leaves the room. The nurse realizes
the behavior and mannerism of Mr. Principe reminds him of his strict disciplinarian father who abused him physically.

66. The recognize that his feeling for the client is known as:
A. denial B. counter transference C. revenge D. transference

67. Seeing that his negative feelings for Mr. Principe could affect his nursing care, the nurse applies the concept of
therapeutic use of self when:
A. the nurse talks about his personal feelings towards the client
B. the nurse suppresses his feelings and continue to take care of the client
C. the nurse uses his awareness and asks to be reassigned to another client
D. the nurse uses self-awareness to manage his feelings and thoughts towards the client

68. Mr. Principe is terminally ill and his family is coping with his impending death. The nurse has to deal with his own
thoughts and personal feelings about death and grieving in order to:
A. avoid sharing personal thought about their impending loss and feeling of grief since this is very subjective
B. get self out of the way while he assists the client and his family express their feelings of impending loss
C. prevent self from being affected by the family’s grief and remain objective
D. help the family plan for the funeral arrangement and burial services

69. One morning when the nurse enters the client’s room. Mr. Principe asks the nurse to “leave me alone and stop
bothering me and I don’t want your pity”. The following response by the nurse would be MOST appropriate?
A. “You seem upset this morning” and remains with the client
B. “You are probably upset because you don’t feel well”
C. “Why you are angry with me? What did I do anything to upset you?
D. “I understand and will leave you for a while”

70. The nurse understands that the nurse-client relationship is a therapeutic alliance when:
A. the nurse is a role model for a client
B. this is an essential part of the nursing process
C. the nurse has to be therapeutic at all times
D. how the nurse thinks and feels affects her actions and behavior towards her client and her work

Situation 15: Mr. Ong is for admission to the medical unit and you are his nurse.

71. The MOST important initial nursing approach when admitting client is to:
A. introduce the client to the ward staff
B. orient the client to the physical set up of the unit
C. identify the most immediate needs of the client and implement the necessary intervention
D. take V/S fro baseline assessment

72. When gathering baseline data, the BEST way for you to check if the client has pedal edema is to:
A. talk to the relatives C. do auscultation
B. interview the client D. do a physical assessment

73. You want to know the sleeping pattern of Mr. Ong You will:
A. interview the clients and relatives
B. take his BP before sleeping and upon waking up
8
C. observe his sleeping pattern over a period of time
D. perform physical assessment

74. Mr. Ong has severe pedal edema. Which accessory device would be appropriate for his condition?
A. footboard B. cradle C. bed board D. rolled pillows

75. A student nurse is observed putting a standard size cuff on an obese client. The action would probably result in BP
reading that is:
A. false high B. false low C. normal D. undetectable

Situation 16: Health is wealth specifically in this time of the century. The nurse is trained to promote well being of the
people.

76. How does a nurse promote one’s well being?


A. periodic travels for rest and recreation
B. faithful and observance of healthy simple lifestyle
C. run away from polluted, stressful areas
D. avoid sleepless, over fatigue nights

77. The nurse can be involved with health promotion as a significant person in helping the family:
A. become a better family C. control their symptoms
B. prevent disease D. modify health promotive behaviors

78. The nurse should NOT leave medication at the bedside because:
a. the bedside table is not sterile
b. it is convenient for the nurse
c. the nurse will not be able to accurately document that the patient actually took the
medication
d. the patient may forget to take it.

79. Non-pharmacologic pain management includes all the following EXCEPT:


a. relaxation techniques c. use of herbal medicines
b. massage d. body movement

80. When assessing a client’s blood pressure, the nurse finds it necessary to recheck the reading. How many seconds
after deflating the cuff should the nurse wait before rechecking the pressure?
a. 10 b. 30 c. 45 d. 60

Situation 17: Safe nursing practice involves an understanding of the law.

81. In the Philippines, this law is :


a. The Philippine Nursing Act of 2002 or R.A. 9173
b. the Philippine Nursing Act of 1991 or R.A. 7164
c. IRR or Resolution 425 of 2003
d. Republic Act No. 8981

82. The Philippine Nursing Act delineates the scope of nursing. It specifies that independent practicing nurse is
responsible for:
a. health promotion and prevention of illness
b. administration of written prescription for treatment and therapies
c. rehabilitative aspect of care
d. Collaborating with other healthcare providers for health restoration and alleviation of suffering

83. Standards of care provide the legal basis for evaluation of nursing practice or malpractice. Its functions include all
EXCEPT:
a. used by nurse experts to define what appropriate nursing practice is in a given situation
b. used to measure or evaluate nursing conduct to determine if the nurse acted reasonably as any prudent
nurse would under similar circumstances
9
c. used to delineate the scope, function and role of the nurse
d. use to measure or evaluate the conduct of nurse specialists who are certified in their own
specialty fields

84. As a standard in ethics, this represents an understanding and agreement to respect another person’s right to
decide a course his or her own destiny:
a. Autonomy c. Beneficence
b. justice d. nonmalifecence

85. The Code of Ethics refers to standards of behavior or ideals of conduct. The ability to answer for and stand by
one’s action refers to:
a. accountability c. advocacy
b. veracity d. responsibility

Situation 18: An understanding of the factors influencing the health care delivery system will enable nurses to
adjust to change, create better ways of providing nursing care and develop new nursing roles.

86. Wellness clinics and health education activities have been integrated in government hospitals to render
appropriate services. Which of the following purposes LEAST helps clients in cases of these health promotion
activities?
A. maintain maximum functions C. promote health habits
B. reduce the costs of health care D. identify disease symptoms

87. With regards to illness prevention activities as part of nursing care, which of the following will help clients MOST?
A. maintain maximum function C. promote habits related to health care
B. reduce risk factor D. manage stress

88. By experience, which of the following nursing goals are MOST often overlooked by nurses and other members of
the hospital team in the care of their clients in the hospital?
A. illness prevention C. diagnosis and treatment
B. health promotion D. rehabilitation of patients

89. Which of the following health care agencies is usually family-centered, relatively recent in popularity and
oftentimes focuses on maintenance of comfort and satisfactory lifestyle of clients in the terminal phase of illness?
A. non-government organization C. community health center
B. hospice D. support group

90. Which of the following is NOT a legally binding document but nonetheless very important in the care of all
patients in any setting?
A. Bill of rights as provided in the Philippine Constitution
B. Scope of nursing practice as defined in R.A. 9173
C. Patient’s Bill of Rights ( as adopted by American Nurses Association )
D. Board of Nursing resolution adopting the Code of Ethics

Situation 19: One of the professional competencies that nurse must always demonstrate is in the area of
communication:

91. Which communication technique would be MOST effective in eliciting detailed information from the client?
A. open-ended questioning C. active listening
B. verbalizing observations D. summarizing clients views

92. Which of the following terms refer to the sender’s attitude towards the self, the message and the listener?
A. verbal communication C. non-verbal communication
B. double-bind communication D. meta communication

93. In interacting with patients, the nurses should remember that a client’s personal space is:
A. that which revolves around the client
B. highly mobile depending upon certain situation
10
C. clearly visible to others
D. the same as that of the nurse

94. In interpersonal communication is LEAST threatening during what type of relationship?


A. social C. personal
B. intimate D. professional

95. In demonstrating the method for deep breathing exercises, the nurse places the hands on the client’s abdomen to
explain diaphragmatic movement. This technique involves the use of which element of communication?
A. appropriateness C. channel
B. feedback D. message

Situation 20: Roy, an adolescent, was diagnosed to have pneumonia. He constantly complains of chest pain and
has a standing order of Morphine SO4.

96. Which of the following MOST appropriately describe pain sensation that has periods of remission and
exacerbation?
A. chronic C. acute
B. intractable D. Psychosomatic

97. Roy is constantly asking to be relieved from pain. Since morphine is an addicting drug, which of the following is
BEST for the patient?
A. administer morphine SO4 PRN
B. administer morphine on a routine schedule as ordered
C. give instructions on relaxation technique to reduce frequency of pain sensation
D. divert the attention by not limiting visitors

98. To get accurate information about the quality of pain the patient is experiencing, which of the following
statements would be MOST APPROPRIATE?
A. “What cause you the pain?” C. “Have you taken something to relieve the pain?”
B. “ Tell me what your pain feels like” D. “Is it stubbing or radiating pain?”

99. As the nurse assigned to Ray, which of the following can decrease his chest pain?
A. supporting his rib cage when he coughs C. teaching him
B. advising him D. encouraging him to breathe deeply

100. Which of the following is the nurse’s primary goal in caring for clients with chronic pain?
A. change the clients perception of pain
B. reduce the clients perception of pain
C. change the clients reaction to pain
D. enumerate the source of pain
-----------------------------------------------------------------------------------------------------------------------------------------------------------
----

SET A Seat No.____


---------------------------------------------------------------------------------------------------------------------------------
NURSING PRACTICE B:
COMMUNITY HEALTH NURSING
AND CARE OF MOTHER AND CHILD

DIRECTION: Choose the letter of the BEST answer by shading the corresponding letter of your choice on the answer
sheet provided. STRICTLY NO ERASURE!

1. A child who is 13 months has fast breathing if he has:


A. 30 breaths per minute or more

11
B. 60 breaths per minute
C. 40 breaths per minute or more
D. 50 breaths per minute

2. Which of the following is true about discharge planning?


A. basic discharge plans involve referral to community resources
B. discharge plans involve referral to community resources
C. simple referral involves use of a discharge planner
D. complex referral includes interdisciplinary collaboration

3. A nurse may keep opened vials of OPV for use in the next session if:
A. you have taken out the vaccine at the health center for some other reasons
B. the expiry date has not passed
C. the vaccines have been stored at a temperature between 0 deg C and 8 deg C
D. the vaccines have change its color

4. The primary preventive measures against HIV-AIDS is:


A. withdrawal C. foams and gels use
B. virus killing drugs D. condom use

5. Ms. Amodia, RN works for a home health agency and cares for an older adult mental patient. On reporting to work,
she observes numerous bruise and red marks on her patients face. Her patient seems upset. Ms. Amodia suspects
that either abuse has occurred. Ms. Amodia should:
A. contact the mobile police to investigate the abuse
B. make an appointment with an elder adult counselor
C. wake her son and ask him who would hurt his mother
D. call her supervisor right away to report the findings

6. If there develops severe pre-eclampsia, what would be the drug you would anticipate to be prescribed and that
you would have to administer?
A. a loop diuretic C. ranitidine (zantac)
B. magnesium sulfate D. a non-steroidal inflammatory agent

7. As a community health nurse you always bear in mind that the purpose of empowering victims of violence is to:
A. tell them how to solve their problems
B. convince victims to leave their abusers
C. help clients become aware that they have control over their lives
D. develop safety escape plans for them

8. Maria is developing constipation from being on bed rest. What measure would you suggest she take to help
prevent this?
A. drink 8 full glasses of fluid such as water daily
B. drink more milk, increased calcium intake prevents constipation
C. eat more frequent small meals instead of three large ones daily
D. walk for at least half an hour daily to stimulate peristalsis

9. A bilateral amputee is assisted by his wife and children to the commode for bowel evacuation. This example best
demonstrates the family’s assistance to meet which need?
A. Elimination C. nutrition and metabolism
B. activity D. health perception and health maintenance

10. Which of the following is the most important reason for doing a literature review for constructing a research
study?
A. helpful information on demographic instrument development could be uncovered
B. existing knowledge about the identified problem can be found
C. a determination of the study’s feasibility could be extrapolated
D. the research design can be copied from another study

12
11. Martina develops endometritis. What would be the best activity for her?
A. lying in bed with a cold cloth on her forehead
B. reading while resting in a trendelenburg position
C. sitting with her feet elevated while playing cards
D. walking around her room listening to music

12. Grace sustained a laceration on her leg from automobile accident. Why are lacerations of lower extremities
potentially more serious among pregnant women than other?
A. lacerations can provoke allergic responses due to gonadotropic hormone release
B. a woman is less able to keep the laceration clean because of her fatigue
C. healing is limited during pregnancy so these will not heal until after birth
D. increased bleeding can occur from uterine pressure on leg veins

13. Fely has diarrhea for 2 days. She has sunken eyes, skin pinch goes back very slowly, and she is drinking poorly
and irritable. She is not able to drink and there is no blood in the stool. How will you classify Fely’s illness?
A. persistent diarrhea C. severe dehydration
B. some dehydration D. no dehydration

14. The infectious agent that causes pulmonary tuberculosis is:


A. mycobacterium tubercle C. Wuchereria bancrofti
B. Hansen’s bacillus D. mycobacterium diphtheria

15. Your client, who happens to be female resident of the barangay you are covering, is an adult survivor who states:
“Why couldn’t I make him stop the abuse? If I were stronger person, I would have been able to make him stop.
Maybe it was my fault to be abused”. Based on this, which would be your most appropriate nursing diagnosis?
A. social isolation C. chronic low self-esteem
B. anxiety D. ineffective family coping

16. Maybelle is also scheduled to have an amniocentesis to test for fetal maturity. What instructions would you give
her before this procedure?
A. void immediately before the procedure to reduce your bladder size
B. no more amniotic fluid forms afterward, that is why only a small amount is removed
C. the intravenous fluid infused to dilate your uterus does not hurt the fetus
D. the x-ray used to reveal your fetus position has no long-term effects

17. When planning teaching strategy for a pregnant woman, the nurse should do which of the following?
A. give information about how the woman can manage the specific problems she identifies as relevant in her
life
B. omit information related to minor pains of pregnancy to prevent the woman from developing hypochondria
C. provide all information to the woman in a group session with other pregnant women so she can have
someone
to discuss it with
D. during the first prenatal visit, teach a woman the care measures necessary for health
promotion
throughout the pregnancy

18. Which of the following symptoms is LESS commonly noted in EARLY pregnancy?
A. frequency of urination C. varicosities
B. chills and fever D. Braxton Hicks’ Contractions

19. Bonnie, 3 months pregnant, has reported for her first prenatal visit. The nurse should instruct her to do which of
the following?
A. eat more dairy products and green leafy vegetables to provide an additional 300 calories each
day
B. increase her intake of carbohydrates-breads and sweets to prevent protein metabolism
C. eat whenever she feels hungry because her body will let her know when she needs nutrients and extra
calories
D. limit intake of amino acids to prevent development of diabetic ketoacidosis
13
20. The nurse is assisting Mr. and Mrs. Cruz to prepare for childbirth in the home setting. Which of the following
supplies should be readily available for the infant immediately after birth?
A. vitamin K to avoid bleeding
B. mild soap without perfume so as not to irritate the skin
C. bulb syringe to suction fluid and mucous from the mouth
D. heating lamp to avoid chilling from water evaporation

21. Which of the following could be included in the outcome criteria for a patient with a nursing diagnosis,
“Knowledge deficit related to potential for altered tissue perfusion in fetus or mother related to maternal
cardiovascular disease”?
A. bedrest is maintained at home after the 36th week of gestation
B. fetal heart rate will remain between 120 and 160 beats a minute
C. jugular vein distention is evident when lying at 45 degrees
D. maternal blood pressure maintained above 150 systolic

22. Pregnant women should be taught to be careful to avoid accidental injury. They are prone to falls for which of the
following reasons?
A. additional weight from pregnancy may disturb balance when walking
B. fetal activity stimulates the nerves of the legs and causes weakness
C. high levels of hormones often impair judgment resulting in reckless behavior
D. increased adrenalin released during pregnancy causes women to move faster than usual

23. When a nurse uses the IMCI model, the IMCI chart uses illness classification, e.g. the pink row needs:
A. no specific treatments such as antibiotics C. appropriate antibiotics
B. urgent referral D. no urgent measures

24. Which of the four signs of good attachment is true in this statement?
A. the chin should touch the breast while the mouth is wide open and while the lower lip is turned inward
more areola is visible above than below
B. the chin should touch the breast, the mouth is wide open while the lower lip turned outward
and more areola visible above than below
C. the chin should touch the breast while the mouth is wide open while the lower lip turned outward and
more areola visible below than above
D. the chin should touch the breast while the mouth is wide open and the lower lip turned inward, more
areola is visible above than below

25. Which type of research inquiry investigates the issues of human complexity ( e. g. understanding the human
expertise?)
A. positivism C. logical position
B. quantitative research D. natural inquiry

26. Carol is 15 months old and weighs 5.5 kgs and it is her initial visit. Her mother says that Carol is not eating well
and unable to breastfeed, he has no vomiting, has no convulsion and not abnormally sleepy or difficult to awaken.
Her temperature is 38.9 deg C. Using the integrated management of childhood illness or IMCI strategy, if you were
the nurse in charge of Carol, how will you classify her illness?
A. a child at a general danger sign C. severe pneumonia
B. very severe febrile disease D. severe malnutrition

27. Why are small for gestational age newborns at risk for difficulty maintaining body temperature?
A. their skin is more susceptible to conduction of cold
B. they are preterm so are born relatively small in size
C. they do not have as many fat stored as other infants
D. they are more active than usual so they throw off comes

28. Oxytocin is administered to Rita to augment labor. What are the first symptoms of water intoxication to observe
for during this procedure?
A. headache and vomiting C. a high choking voice
14
B. a swollen tender tongue D. abdominal bleeding and pain

29. Which of the following treatment should NOT be considered if the child has severe dengue hemorrhagic fever?
A. use plan C if there is bleeding from the nose or gums
B. give ORS if there is skin Petechiae, persistent vomiting, and positive tourniquet test
C. give aspirin
D. prevent low blood sugar

30. In assessing the patient’s condition using the Integrated Management of Childhood Illness approach strategy, the
first thing that a nurse should do is to:
A. ask what are the child’s problem C. check for the four main symptoms
B. check the patient’s level of consciousness D. check for the general danger signs

31. A child with diarrhea is observed for the following EXCEPT:


A. how long the child has diarrhea C. presence of blood in the stool
B. skin Petechiae D. signs of dehydration

32. The child with no dehydration needs home treatment. Which of the following is NOT included in the care for
home management at this case?
A. give drugs every 4 hours C. give the child more fluids
B. continue feeding the child D. inform when to return to the health center

33. Ms. Jordan, RN, believes that a patient should be treated as individual. This ethical principle that the patient
referred to:
A. beneficence C. respect for person
B. nonmaleficence D. autonomy

34. When patients cannot make decisions for themselves, the nurse advocate relies on the ethical principle of:
A. justice and beneficence C. beneficence and nonmaleficence
B. fidelity and nonmaleficence D. fidelity and justice

35. Being a community health nurse, you have the responsibility of participating in protecting the health of people.
Consider this situation: Vendors selling bread with their bare hands. They receive money with these hands. You do
not see them washing their hands. What should you say/do?
A. “Miss, may I get the bread myself because you have not washed your hands”
B. All of these
C. “Miss, it is better to use a pick up forceps/ bread tong”
D. “Miss, your hands are dirty. Wash your hands first before getting the bread”

36. If a child with diarrhea has 2 or more signs in the IMCI pink row chart, the classification will be:
A. moderate dehydration C. no dehydration
B. some dehydration D. severe dehydration

37. If the capillary refill takes more than 3 seconds it may mean:
A. the child is in shock C. circulatory failure
B. the child is dehydrated D. the child is alright

38. Leprosy is a chronic disease of the skin and peripheral nerves. Which of the following signs are present in the
early stage?
A. loss of eyebrows C. contractures
B. clawing of fingers D. thickening or painful nerves

39. In asking the mother about her child’s problem the following communication skills should be used EXCEPT:
A. use words that the mother understand C. listen attentively
B. give time for the mother to answer the question D. ask checking questions

40. Patients eligible under the short term chemotherapy (STC) are the following EXCEPT:
A. all newly discovered and reconfirmed sputum positive cases
15
B. those with cavitary lung findings after two successive negative sputum examination
C. patients without having been given consent for anti-TB treatment
D. patients willing to undergo treatment

41.When planning comfort measures to help the woman in active labor to tolerate her pain, the nurse must consider
which of the following?
A. early labor contractions are usually regular, coordinated, and very painful
B. if women are properly prepared, they will require no pain medication to manage their pain
C. pain medication given during the latent phase of labor is not likely to impair contractions
D. the acceleration phase of labor can be a time of true discomfort and high anxiety

42. Francis is admitted in active labor. The nurse locates fetal heart sounds in the upper left quadrant of the mother’s
abdomen. The nurse would recognize which of the following?
A. Francis will probably deliver very quickly and without problems]
B. This indicates Francis will probably have a breech delivery
C. The fetus is in the most common anterior fetal positions
D. this position is referred to as being left anteriopelvic

43. While interviewing a woman in labor, the nurse would address which of the following?
A. whether the pregnancy was planned C. maternal concerns regarding fetal health
B. the use of medications during pregnancy D. all of the above

44. If a woman will be placing her baby for adoption, which of the following nursing measures should be
implemented during the labor stages?
A. avoid discussing the baby during the historical assessment to minimize the woman’s anxiety
B. support the woman as needed by accepting the decisions she makes regarding holding the
baby
C. protect the woman from visitors and family members who might try to change her mind
D. take the baby away as soon as possible after birth to prevent bonding from occurring

45. During the third stage of labor, the nurse may have which of the following responsibilities?
A. administration of intramuscular Oxytocin to facilitate uterine contractility
B. monitoring for blood loss greater than 100 cc, which would indicate gross hemorrhage
C. noting if the placenta makes a Schultz presentation, which is a sign of gross complication
D. pushing down on the relaxed uterus to aid in the removal of the placenta

46. Immediately following episiotomy repair, the nurse would do which of the following?
A. cleanse the woman’s anal area, then perineum and vulva, to remove any fecal incontinence or
vaginal secretions
B. monitor the woman for shaking and complaints of chill sensations, which may indicate an adverse reaction
to medications
C. palpate the uterus fundus for size, consistency and position and take vital signs to obtain baseline data
D. remove all coverings except a clean, light hospital gown to prevent the development of postpartal fever

47. Mr. Tony has been hospitalized for months following special spinal cord surgery. The boys club in his hometown
was renamed the Tony boys club, and a parade is planned to honor Mr. Tony. Such community action should
positively influence Mr. Tony’s:
A. self-perception and self concept C. health perception and health maintenance
B. cognition and perception D. coping and stress tolerance

48. Which of the following conditions is not true about contraindication to immunization?
A. do not give BCG if the child has known hepatitis
B. do not give BCG if the child has known AIDS
C. do not give DPT to a child who has recurrent convulsion or active neurologic disease
D. do not give DPT2 or DPT3 to a child who has had convulsions within 3 days of DPT1

49. Which of the following might prevent the conduct of a research study?
A. in order to gather data for the research, excessive risk to subject is required
16
B. costs for conducting the study are low compared to the potential benefits from the researcher
C. the identified problem is covered in the literature
D. the research problem is based on untested nursing theories

50. Marsha is concerned she may loose an excessive amount of blood with cesarean surgery. What is the usual
amount of blood loss with cesarean birth?
A. 250 – 350 ml C. 100 – 220 ml
C. 500 – 1000 ml D. 300 – 500 ml

51. If the child is 4 months, which of the following questions SHOULD NOT be included in checking the general
danger signs?
A. ask if the child is eating well during illness C. ask if the child has had no convulsion
B. ask if the child is able to breastfeed D. ask if the child vomits everything

52. Arnie, like all newborns, can loose body heat by conduction. Under which conditions is this most apt to occur?
A. if the nursery is cooled by air conditioning C. if Arnie is placed in a cold bassinet
B. if the infant is wet from amniotic fluid D. if there is a breeze from an open window

53. You have just entered Mr. Tiangco’s room and observed him lying on the floor next to the bed. The side rails on
the bed are down. When you asked Mr. Tiangco what happened, he replies. “I was asleep and the next thing I knew
I was on the floor”. Which of the following examples of documentation is MOST APPROPRIATE for this situation?
A. Lying on floor next to bed. No complaints verbalized. Side rails down. Appears to have fallen out of bed
while asleep.
B. Mr. Tiangco was found lying on the floor next to his bed. When asked what happened, he
stated “ I was asleep and the next thing I knew I was on the floor”.
C. Lying on the floor, side rails down. Rolled out of bed while asleep.
D. Mr. Tiangco fell out of bed while asleep; both side rails were left down.

54. The application of public health, medical and engineering practices to health services and effectiveness of workers
may be termed as:
A. community health C. school health
B. occupational health D. mental health

55. A literature review should be:


A. brief and limited C. systematic and exhaustive
B. broad and theoretical D. general and context

56. The Santiago Family lives in a makeshift house in Julugan, Tanza, Cavite. Composed of 8 members, the eldest
son Rhio is 7 years old. Rhean is 6, Queenie and Quency are twins 5, Peter is 4 and the youngest is 6 months old
named Oscar. Aling Rosa works in a laundry earning Php 1,500.00 a month and husband Mang Sony works as fish
vendor earning Php 150.00 per day. Peter and the twins are enrolled in the day care. Rhio and Rhean stopped
studying to help their parents for their younger brothers and sisters. With the current condition besetting the
Santiago family, which of the following interventions can lend assistance to them?
A. refer the children to a hospice care facility
B. assist Aling Rosa to be referred to the DSWD for livelihood assistance
C. give Aling Rosa’s family a Php 2,000.00 worth of capital for them to put a fishball stand
D. advise Aling Rosa to use contraceptive pills

57. Every year we discover new methods or gadgets to improve man’s life. The research typed used of this nature is:
A. applied C. experimental
B. developmental D. quantitative

58. When should the mother give complementary foods to a 5 months old infant?
A. if the child gives adequate weight for his age
B. if the child shows interest in semi solid foods
C. supplementary foods should be given before breastfeeding
D. if the child is breastfed less than 8 times in 24 hours

17
59. Hepatitis A is differentiated from hepatitis B by their mode of transmission. Hepatitis B is transmitted through:
A. insect bites C. transfusion and injection
B. urine D. fecal waste

60. The nurse wants to immunize a 1 year old child for measles. The child has been classified as having pneumonia
and no anemia and not very low weight. The child’s mother does not want her child to be immunized and instead
they will just return as soon as the child is better. How will the nurse explain the importance of immunizing the child
with measles now?
A. the child’s condition is not a hindrance to immunization according to W.H.O.
B. the child has lower resistance and more prone to disease
C. the child will recover faster if the child will be immunized
D. it is the child’s schedule to have the immunization

61. To implement the case finding aspect of TB control, sputum examination should be done to the following groups:
A. children 0-9 years old with cough for 2 weeks or more
B. all persons of all ages above 1 year with symptoms indicative of tuberculosis
C. children below 10 years old with fever for 3-5 days
D. all persons ages 10 years and over with progressive loss of weight

62. Mr. Barmonte was brought home with an advance directive, but the nurse is not sure that she can follow his
wishes. The nurse should:
A. follow the directive even though the nurse is uncomfortable with the directives
B. call Mr. Barmonte’s lawyer
C. discuss with the interdisciplinary team in charge of Mr. Barmonte and the organization’s
ethics committee
D. ignore the advance directive

63. A term concerning body resistance which refers to protein present in the serum of the blood:
A. antibodies C. antigen
B. antitoxin D. allergen

64. A health care service in which risk factors are identified, occupational safety measures reduce, and a public
education program begun is:
A. health promotion C. treatment facility
B. illness prevention D. rehabilitation

65. A community health nurse (CHN) visits the Gomez family weekly. Although family income seems adequate, Mrs.
Gomez is unable to budget it over a 4-week period. The nurse asks what may be done to help Mrs. Gomez shrugs her
shoulders and says, “Tomorrow may never come.” Mrs. Gomez’s reaction is described as:
A. reflecting her culture, values, and time orientation
B. one of not caring about her family’s needs
C. her belief that income may not be adequate as perceived by the CHN
D. indicating that her budget needs is a private matter

66. A positive or negative feeling toward a person, object, or idea is known as a/an:
A. merit C. moral
B. value D. attitude

67. When the nurse assists the clients in understanding personal values, this is an example of value:
A. validation C. clarification
B. discovery D. choice

68. While doing a nutritional assessment of a low-income family, the community health nurse determines the family’s
diet is inadequate in protein content. The nurse could suggest which of the following foods to increase protein
content with little increase in food expenditure?
A. oranges and potatoes C. rice and macaroni
B. potatoes and rice D. peas and beans
18
69. A new mother who is breast-feeding her baby asks the nurse when she should start her feeding baby vegetables.
The nurse would suggest the MOST APPROPRIATE age to begin vegetables is:
A. 3 to 4 months C. 6 to 8 months
B. 4 to 5 months D. 9 to 12 months

70. Assessment areas for the nurse is working with the family on health promotion strategies would include:
A. the television shows that they watch C. the perceived health status and illness patterns of
the
family
B. the family and all the relative’s statuses D. the mental status of family and friends

71. The nurse manager wishes to implement a new way of determining the vacation schedule for the staff. The
senior staff opposes the change while the newer staff seems more accepting of the change. An effective strategy for
resolving this difference in acceptance would be to:
A. explain that the change will occur as designed, regardless of the staff’s preference
B. tell the staff that if they really do not want the change, it will not be implemented
C. provide extensive and detailed rationale for the proposed change
D. encourage each side to share their views with each other

72. The following are the duties of the Public Health Nurse EXCEPT:
A. leave to the BHW the responsibilities of educating the community
B. provision of nursing care to the sick and well individuals
C. provision of technical and administrative support to rural health midwives
D. conduct of pre and post consultation conference for clinic patients

73. An 8 month old has fast breathing if he has:


A. 40 breaths per minute C. 38 breaths per minute
B. 55 breaths per minute D. 45 breaths per minute

74. The following are the concerns of the Public Health Nurse on the third trimester of pregnancy EXCEPT:
A. advice for the mother to take oral contraceptive
B. where to give birth
C. proper timing in the separation of mother to other siblings
D. type of feeding

75. When a child with persistent diarrhea returns for follow-up visit after 5 days, the nurse should ask the mother if
the diarrhea has stopped when:
A. the child has less than 3 loose bowels per day
B. the child has 4 semi formed stool
C. the child has 5 formed stools
D. the child has 3 loose stools/day

76. In barangay Y, one of the identified problems is unavailability of toilet facility for the residents. The appropriate
intervention is:
A. secure donations in the form of toilet bowls
B. ask help from local government
C. none of the above
D. conduct a community assembly and discuss the consequences of this problems

77. A home visit is a professional interaction between the community health nurse and his patient or the family.
Where could the nurse get available information about the patient or the family to be visited?
A. document file C. doctor’s office
B. records and reports D. family health record

78. In planning a home visit, the nurse considers the individual needs. Which of the following is a priority?
A. needs of all family members C. follow-up of medication and treatment
19
B. needs recognized by the family D. clinic schedules after home visit

79. Which of the following is the FIRST action of the nurse during a home visit?
A. greet the client or the household members
B. state the purpose or objective of the visit
C. give necessary health teaching
D. inquire about welfare and health condition of the client

80. Which factor must be considered by the nurse as vital to determine frequency of the home visit?
A. available resources C. acceptance of the family
B. policy of the agency D. past nursing services

81. Eating habits of the family has changed due to the existing fast food establishment in the area. What health risk
should you warn the family?
A. food-borne infection C. increase weight
B. indigestion D. hand washing

82. To show a mother how to help her to have good attachment of the infant during breast feeding. Which of the
following statement SHOULD NOT be included?
A. place the infant in your most convenient position
B. touch the infant’s lips with her nipples
C. wait until the infant’s mouth is widely open
D. move the infant quickly onto her breast, aiming the infant’s lips well below the nipple

83. When members of the community identify needs, work on its solutions with confidence in the spirit of
cooperation. The process is:
A. nurse’s professional effort C. community organizing
B. government assistance D. leadership through traditional leaders

84. In assessing the level of family cohesion, you have determined that the family is very close and has very high
loyalty, and members are highly dependent on one another. You would document this as which level of cohesion:
A. separated C. connected
B. enmeshed D. disengaged

85. Mrs. Ayuyao, 77 year old, has been admitted with pneumonia. Her husband asks the nurse about the living will.
As a license nurse, you remember that living wills:
A. are legally binding in all states
B. allow the court to decide when the care can be given
C. allow the individual to express his or her wishes regarding care
D. allow health workers to withhold fluids and medications

86. Maybelle is scheduled to have an ultrasound examination. What instructions would you give her before her
examination?
A. The intravenous fluid infused to dilate your uterus does not hurt the fetus
B. you will need to drink at least 3 glasses of fluid before the procedure
C. void immediately before the procedure to reduce your bladder size
D. you can have medicine for pain for any contractions cause by the test

87. What is the most common type of health problem seen in the health care system?
A. poor prenatal care C. immobility
B. lack of information available to patients D. increased rate of chronic disease

88. Which of the following is the best type of disease prevention?


A. immunization C. community social events
B. yearly physical exam D. behavior that promotes health

89. Nurses who deliver community-based care must have which of the following qualities?
A. adaptability C. ability to be self-directed
20
B. tolerance of various lifestyle D. all of the above

90. A nurse has scheduled a hypertension clinic. This service would be an example of which of the following types of
health care?
A. tertiary prevention C. primary prevention
B. secondary prevention D. protection

91. When providing care in a home, how might the nurse best implement infection control?
A. cleanse the hands before and after giving direct patient care
B. remove the patient’s wound dressings from the home
C. dispose off patient’s syringes in the patient’s garbage
D. disinfect all work areas in the patient’s home

92. Which of the following is a leading health problem of school-age children?


A. earaches C. obesity
B. attention disorders D. malnutrition

93. If the child does not have ear problem, what should the nurse do?
A. check for tender swelling behind the ear C. check for ear drainage
B. check for ear pain D. got to diet question, check for malnutrition

94. Nona had her pelvic measurement taken. What size should the ischial tuberosity diameter be, to be considered
adequate?
A. have the width of the symphysis pubis C. 11 cm
B. 8 cm D. twice the width of the conjugate diameter

95. In classifying the child’s illness using the IMCI color-coded triage, each illness is classified according to whether it
requires the following EXCEPT:
A. urgent pre-referral treatment and referral C. immunization
B. simple advice on home management D. specific medical treatment

96. Which of the following is an example of hostile environment in terms of sexual harassment?
A. the boss assures you of a big promotion if you go out on a couple of dates with him
B. your supervisor makes masturbatory gestures every time you walk pass him
C. the personnel manager hints that the job will be yours if you cooperate sexually with him
D. your boss suggests that your “raise” is dependent upon having sex with him
97. Which among these activities is the concern of the Primary Level?
A. administration of BCG vaccine for a 10 days old baby
B. insertion of a nasogastric tubes to patients
C. administer diuretics such as furosemide per doctors order
D. regulate intravenous fluid drop rates

98. Which of the following vaccine in the Expanded Program of Immunization have a 4 weeks interval?
A. OPV C. BCG
B. AMV D. Tetanus Toxoid

99. The nurse who is planning a health promotion program with clients in the community will have the LEAST focus
on:
A. assisting the clients to make informed decisions
B. organizing methods to achieve optimal mental health
C. reducing genetic risk factors for illness
D. providing information and skills to maintain lifestyle changes

100. A holistic belief system by the nurse would be most evident if the nurse:
A. incorporates client perceptions of health when planning care
B. encourages behavior modification program
C. supports goal-directed learning to improve health
D. accepts death as an outcome of life
21
---------------------------------------------------------------------------------------------------------------------------------------------------------------
------

SET A Seat
No.____
---------------------------------------------------------------------------------------------------------------------------------
NURSING PRACTICE 3A:
NURSING CARE OF CLIENT WITH PHYSIOLOGICAL
AND PSYCHOSOCIAL ALTERATIONS
DIRECTION: Choose the letter of the BEST answer by shading the corresponding letter of your choice on the answer
sheet provided. STRICTLY NO ERASURE!

1. A group of nurses are studying the case of a 16-year old nursing student who was accused of frustrated
homicide. Benny can determine their understanding of circumstantial evidence if they can identify which of the
following in the case?
a. justifying circumstances c. exempting circumstances
b. aggravating circumstances d. mitigating circumstances

2. E.M. is a 30-year old premenopausal female who is asking the nurse the most appropriate time of the month to
do her self-examination of the breast. The MOST appropriate reply by the nurse would be:
a. the 26th day of the menstrual cycle
b. 7 to 8 days after conclusion of the menstrual period
c. during her menstruation
d. the same day each month

3. Copies of certain portions of the chart maybe released at the discretion of the hospital but may be limited to:
a. x-ray result, treatment orders, and surgery technique
b. laboratory results and abstract only
c. laboratory results, OR reports and patient’s abstract
d. OR reports, laboratory results, nurse’s notes

4. Jenny is reading the progress notes of her patient and reads that the physician had documented “insensible fluid
loss of approximately 800 ml daily.” She understands that this type of fluid loss can occur through:
a. urinary output c. the skin
b. the gastrointestinal part d. wound drainage

5. Communication can be verbal and non-verbal. Examples of non-verbal communication in patient-nurse relationship
are the following EXCEPT:
a. signs c. symptoms
b. restlessness d. using an interpreter like a family member

6. The perioperative nurse assumes responsibility and accountability for nursing judgments and actions exemplified
by the following, EXCEPT:
a. accepting on-call assignment
b. accepting committee assignment
c. selecting a.m. shift only
d. maintaining basic nursing procedure

7. What one condition besetting the nursing service prevents the use of team approach?
a. fast turn-around of nurses c. understaffing
b. no administrative support d. politics in the nursing department

22
8. Fe, a nurse at the PACU discovered that Luisa, 50 kilos who is 3 hours post cholecystectomy was in severe pain.
Upon checking the chart, she found out that Luisa had “Demerol 100 mg I.M. prn for pain”. What should Fe do?
a. verify the order from the M.D.
b. inject 100 mg. Demerol I.M. to Luisa
c. report to the nurse supervisor for opinion
d. administer the recommended dose which is 50 mg because Luisa weighs 50 kilos

9. Postoperatively, the nurse teaches the patient with a modified radical mastectomy to prevent lymphedema by:
a. avoiding unnecessary trauma (e.g. Venipuncture, BP) to the arm on the operative side
b. using a sling to keep arm flexed at the side
c. exposing the arm to sunlight to increase circulation
d. wrapping the arm with elastic bandages during the night

10. Ms. De Leon is suffering from Bell’s palsy as indicated by a feeling of stiffness and a drawing sensation of the
face. In teaching her about the disease, what would be important to tell her?
a. There may be increased sensitivity to sound
b. There is a heightened awareness of taste, so that foods must be bland
c. The eye is susceptible to injury if the eyelid does not close
d. Drooling from an increase of saliva on the affected side may occur

11. The patient is scheduled for adrenalectomy. In the preoperative period, the priority nursing action would be to
monitor:
a. urine for glucose and acetone
b. vital signs
c. intake and output
d. blood urea nitrogen results

12. Virtue ethics gives us a special prospective in the parent’s role to decide for their children including decision of:
a. professional choice of children
b. spouse choice for children
c. life and death prior to conception and birth
d. gender choice for children

13. One of the reasons why behavior in the OR is so tightly controlled is:
a. it is hard to move around in a OR gown
b. the door of OR suite are closed
c. everybody is busy doing their operation
d. to prevent the cross-contamination of infection between OR staff and patient

14. One of the hidden dangers in the OR is missing instruments. What is the appropriate approach to this happening?
a. correct labeling
b. “a place for everything and everything in its place”
c. install a flush sterilizer in the OR
d. increase instrument inventory

15. Ms. W.O. is found on the floor of her room. She fell while crawling over the side rails of her bed. She is
unconscious and has a large laceration to the head that is bleeding profusely. The nurse’s priority action would be:
a. apply direct pressure to the laceration to her head
b. ensure the patient has an open airway
c. notify the physician
d. check the patient’s vital signs

16. During admission of a patient with a severe head injury to the emergency department, the nurse places the
highest priority on assessment for:
a. presence of a neck injury
b. neurological status with the Glasgow Coma Scale
c. cerebrospinal fluid leakage from ears or nose
d. patency of airway
23
17. In the hospital, the best indicator of quality is:
a. patients recover but spend more on supplies
b. patients come back with referral like their family and friends
c. patients demand more use of technology
d. patients come back of their doctor’s advice

18. A most critical strategy in nursing communication is:


a. non-verbal communication
b. giving stereotyped comments
c. verbal communication
d. active listening

19. Mr. T.O. has undergone surgery for lyses of adhesions. He is transferred from Post Anesthesia Care Unit (PACU)
to the Surgical floor, the nurse should obtain blood pressure, pulse and respiration every:
a. 3 minutes c. 15 minutes
b. 30 minutes d. 20 minutes

20. Mr. K, age 13, is diagnosed with chronic bronchitis. He is very dyspneic and must sit up to breath. An abnormal
condition in which there is discomfort in breathing in any bed or sitting position is:
a. Cheyne-stokes c. eupnea
b. orthopnea d. dyspnea

21. The nurse recognizes that the MOST common causative organism in pyelonephritis is:
a. E.Coli c. Candida Albicans
b. Klebsiella d. Pseudomonas

22. There is a global concern on AIDS. There are legal and ethical concerns specially:
a. duty siblings and spouse
b. need for universal testing
c. confidentiality
d. public support and understanding

23. Mr. WT is cleaning the garage and splashes a chemical to his eyes. The initial priority care following the chemical
burn is to:
a. irrigate with normal saline for 1 to 15 minutes
b. transport to a physician immediately
c. irrigate with water for 15 minutes or longer
d. cover the eyes with a sterile gauze

24. Licensed nurses from foreign countries can practice nursing in the Philippines in the following condition:
a. employed in state colleges and universities
b. special projects with hospitals with a fixed fee
c. employees by private hospitals
d. medical mission whose services are free

25. A patient diagnosed with breast cancer has been offered the treatment choices of breast conservation surgery
with radiation or a modified radical mastectomy. When questioned by the patient about these options, the nurse
informs the patient that the lumpectomy with radiation:
a. reduces the fear and anxiety that accompany the diagnosis and treatment of cancer
b. has about the same 10-year survival rate as the modified radical mastectomy
c. provides a shorter treatment period with a fewer long term complications
d. preserves the normal appearance and sensitivity of the breast.

26. Following a fracture of the forearm or tibia, complaints of sharp, deep, unrelenting pain in the hand or foot
unrelieved by analgesics or elevation of the extremity indicate which complication?
a. cast syndrome b. compartment syndrome c. gangrene d. fat embolism

24
27. Another worthy study is the compliance to the principles of aseptic technique among the sterile OR team. Who
does NOT belong to the sterile OR team?
a. scrub nurse c. x-ray technician
b. assistant surgeon d. surgeon

28. Circulation must be restored within 4 minutes of cardiopulmonary arrest because:


a. the lungs fill with fluid
b. the blood begins to coagulate
c. brain cells begin to die
d. irreversible kidney failure develops

29. Benny elaborated on the concept of informed consent. He determines that the nurses need more explanation if
their response is one of the following:
a. the consent should provide a description of alternative treatments or procedures
b. the consent should offer a thorough explanation of the procedures to be done and the consequences of it.
c. the consent should include the medical diagnosis and explanation of the patient’s condition
d. the consent should describe the prognosis if the recommended care is refused.

30. Which of the assessment findings would indicate a need for possible glaucoma testing?
a. intermittent loss of vision c. halos around lights
b. presence of floaters d. pruritus and erythema of the conjunctiva

31. The diabetic patient asks for a snack and something to drink. The MOST appropriate choice for this client to meet
nutritional needs would be which of the following:
a. crackers with cheese and tea
b. toast with peanut butter and cocoa
c. vanilla wafers and coffee with cream
d. graham crackers and warm milk

32. The patient demonstrates knowledge of the psychological response to the operation and other invasive procedure
when she asks about:
a. Who will be with me in the OR?
b. How is the post operative pain over the site like?
c. Will I be naked during the operation?
d. Is it cold inside the

33. In teaching the mother the proper administration of tetracycline eye ointment, which of the following is MOST
crucial?
a. squirt a small amount on the inside of the infected eye’s lower lid
b. use clean, wet cloth to gently wipe away the pus
c. wash hands before medication administration
d. do not use other eye ointments or drops or put anything else in eyes.

34. The primary goal of nursing interventions after a craniotomy is:


a. avoiding need for secondary surgery
b. ensuring patient comfort
c. preventing increased intracranial pressure
d. prevention of infection

35. Mr. Salvo, a 35 year old male, has been admitted with the diagnosis of peptic ulcers. The nurse recognizes which
drugs as those MOST commonly used in these patients to decrease acid secretions?
a. erythromycin and flagyl
b. Tagamet and zantac
c. Maalox and kayexalate
d. Dyazide and carafate

36. When reading the urinalysis report, the nurse recognizes this result as abnormal:
a. red blood cells 15-20 c. glucose negative
25
b. turbid d. ph 6.0

37. Mr. Hizon has had cataract surgery. Discharge teaching would include:
a. wearing eye patches for the first 72 hours
b. bending at the waist acceptable if done slowly
c. bending at the knees and keeping the head straight
d. lifting light objects is acceptable

38. Endoscopic minimally invasive surgery has evolved from diagnostic modality to a widespread surgical technique.
What department should the nurse collaborate which is unusual in conventional surgery?
a. engineering department c. blood bank services
b. x-ray department d. linen section

39. When the client is discharged from the hospital and is not capable of doing the needed care services, the
following can assume the role, EXCEPT:
a. family members c. significant others
b. chaplain d. responsible caregiver

40. R.N. denotes that a nurse:


a. has satisfactorily completed with the requirements to practice nursing as set by the state.
b. has a baccalaureate degree in nursing
c. is professionally ready to practice nursing
d. just a title

41. Urinary tract infection is the most common site of nosocomial infection particularly with urinary catheterization. It
can be reduced significantly by through:
a. intermittent drainage c. hanging system drainage
b. open system drainage d. closed system drainage

42. The professional license of an RN is:


a. transferable c. can be revoked for reasons stipulated in RA 9173
b. lifetime d. personal

43. In medical and nursing practice, code means a call for:


a. DNR state c. clinical case
b. call to order d. cardiopulmonary resuscitation

44. A thumb mark of a comatose patient in the informed consent is considered:


a. a misrepresentation c. a not valid signature
b. “nothing” d. a valid signature

45. The OR team collaborates from the first to the last surgical procedure. Who monitors the activities of each OR
suite?
a. scrub nurse c. circulating nurse
b. anesthesiologist d. surgeon

46. Organ donation to save life was initially with the first transplant done by:
a. Dr. Christian Barnard of the Union of South Africa
b. Dr. Christian of the U.S.A.
c. Dr. Christian Barnard of Soviet Union
d. Dr. Christian Barnard of U.K.

47. Ms. C.O., age 45, is being evaluated to rule out pulmonary tuberculosis. Which finding is MOST closely associated
with TB?
a. green-colored sputum c. leg cramps
b. night sweats d. skin discoloration

48. Following a renal angiography, the patient assessment priority is the:


26
a. respiratory effort c. urinary output
b. blood pressure d. puncture site

49. In patients with acute pancreatitis, the administration of the analgesic morphine may cause:
a. addiction c. paralytic ileus
b. urinary retention d. spasms of the sphincter of Oddi

50. Who is responsible in daily monitoring the standards of safe, nursing practice in the operating suite?
a. surgeon c. OR nurse supervisor
b. perioperative nurse d. chief nurse

51. Cathy is giving bed bath to an assigned client. A nursing aide enters the patient’s room and informs her that
another patient is in pain needs pain medication. The appropriate action of Cathy is:
a. ask the nursing aide to continue the bed bath after asking the permission of the patient so Cathy can
attend to the other client
b. Cover the client, raise side rails, tell the client that Cathy has to administer the pain
medication first, then will return as soon as possible
c. finish the bed bath, then administer the pain medication
d. ask the nursing aide to stay with the other client while Cathy finishes the bed bath

52. Which of the following health teachings by the nurse will be MOST appropriate in home medication
administration?
a. determine the appropriate drug and dosage for the child’s age or weight
b. ask the mother to give the first dose of the drug to her child
c. demonstrate to the mother how to measure a dose
d. tell the mother the reason for giving the drug to the child

53. The patient’s medical record is the best evidence of the care that is given to the patient. It is the property of:
a. the patient owns the record
b. the physical property of the hospital
c. the health team property
d. the doctor owns the record

54. Edna classified the patient as having diarrhea with severe dehydration. Which of the following interventions would
the nurse employ?
a. advise the mother regarding follow-up after 5 days
b. give a tetracycline tablet
c. refer urgently to the nearest hospital
d. give vitamin A
55. A physician has prescribed propylthiouracil for a client with hyperthyroidism. A nurse develops a plan of care for
the client. A priority nursing assessment to be included in the plan regarding this medication is to assess for:
a. signs of renal toxicity
b. signs and symptoms of hyperglycemia
c. relief of pain
d. signs and symptoms of hypothyroidism

56. An external insulin pump is prescribed for a client with with diabetes mellitus. The client asks Eddie about the
function of the pump. He bases the response on the information that the pump:
a. is surgically attached to the pancreas and infuses regular insulin into the pancreas, which in turn releases the
insulin into the bloodstream.
b. is timed to release programmed doses of regular or NPH insulin into the bloodstream at specific intervals
c. continuously infuses small amounts of NPH insulin into the bloodstream while regularly monitoring blood glucose
levels
d. gives a small continuous dose of regular insulin, and the client can self-bolus with an additional
dosage from the pump prior to each meal.

57. In evaluating the care of Ms. O.P., a young female executive admitted with bleeding peptic ulcer, the nurse
focuses on nursing interventions. A nursing intervention associated with this type of patient is:
27
a. frequently monitoring arterial blood levels
b. checking the patient’s low residue diet.
c. checking the blood pressure and pulse rates each shift
d. observing vomitus for color, consistency, and volume

58. During surgery, movement of personnel should be:


a. kept to a minimum c. monitored
b. restricted d. eliminated when possible

59. Ms. Dee is a 27-year-old housewife and mother of two children. She is being seen by the nurse at the health
maintenance organization for signs of fatigue. She has the history of iron deficiency anemia. Which of the following
data from the nursing history indicates that the anemia is NOT currently managed effectively?
a. pallor c. monitored
b. poor skin turgor d. respirations 18 and regular

60. Which of the following would be MOST effective in promoting adequate respiratory function in an unconscious
client recently admitted to the PACU with no contraindications to movement?
a. extending client’s chin while on his side and pillow at the back
b. turning the client from side to side at 10-minute interval
c. placing the client prone to facilitate drainage of secretions
d. performing jaw thrust maneuver while client is supine

61. The patient has a right to information regarding the operation or other invasive procedure and potential effects.
This right is achieved through:
a. informed consent c. charting
b. preoperative visit d. doctor’s rounds

62. Which statement about a person’s character is evident in the OR team?


a. it assists in the control of feelings, thoughts and emotions in the face of difficulty
b. it reflects the moral values and beliefs that are used as guides to personal behavior and
actions
c. it encourages the constructive use of the pleasure of the senses
d. it refers to the quality of being righteous, correct, fair and impartial

63. Mrs. XP underwent D and C for dysfunctional bleeding. What is inserted vaginally to prevent postoperative
bleeding?
a. perineal pad c. vaginal suppository
b. vaginal packing d. gelfoam

64. Reusable supplies/instruments are still widely used in the Philippine OR setting. A clear advantage of single-use
items is:
a. transmission of infection during processing is avoided
b. convenience
c. guaranteed first class quality
d. integrity and functionality is assured.

65. It is important to teach the patient the warning signs for skin cancer. Which of the following is a warning sign of
cancer?
a. smooth surface c. border irregularity
b. decreasing diameter d. mole symmetry

66. In teaching the patient with Parkinson’s disease, which response would indicate the need for further education?
a. “I should eat a diet high in fiber and roughage to decrease my constipation.”
b. “I need to be sitting straight up and my chin slightly tucked so I won’t choke when I eat or drink.”
c. “I need to exercise at least some everyday.”
d. “If I miss an occasional dose of the medication, it is not of much significance.”

67. It is MOST important to assess the adolescent with acne for:


28
a. change in weight c. suicide tendencies
b. low self esteem d. increase intake of fatty foods

68. Communication can take many forms. Each has its own limitations. Which is a deterrent factor in written
communications?
a. e-mail c. suicide tendencies
b. fax d. use of non-universal abbreviation

69. Monica shared with the interviewer her most recent experiences about a restless pediatric patient whom she puts
up the side rails of the bed to prevent accidental falls. Which of the following attributes is shown by Monica?
a. resourcefulness c. honesty
b. prudence d. reliability
70. As experimental research designs have their strengths, so does non-experimental researches. Which of the
following statements is NOT one of the strengths of non-experimental designs?
a. can be completed in a short span of time
b. less expensive
c. useful for predictive purposes
d. easier to gain cooperation of study subjects

71. Mr. E.O. age 52 had a laryngectomy due to cancer of the larynx. Discharge instructions are given to Mr.F.O. and
his family. Which response by written communication from Mr. F.O. or verbal response from the family, will be a
signal to the nurse that the instructions need to be reclarified?
a. it is acceptable to take over-the-counter medications now that condition is stable
b. the suctioning at home must be a clean procedure, not sterile.
c. report swelling, pain or excessive drainage
d. cleans skin around stoma BID, use hydrogen peroxide and rinse with water, pat dry.

72. Dr. Tuazon scheduled Mrs. Poe for a right breast mass incision with frozen section and possible mastectomy on
Monday, first case. As the nurse in-charge for scheduling you will collaborate with the following departments
EXCEPT:
a. pathology c. anesthesia
b. dietary d. surgery

73. Charting should be legible and include only standard abbreviations. Which of the following is NOT a standard
abbreviation?
a. PRN c. NNO
b. OD d. NPO

74. A mother who is pregnant and has ovarian cancer has to undergo surgery to treat the cancer. In the process the
fetus died. The doctrine that justifies the death of the fetus is:
a. justice c. exception to the role
b. anatomy d. double effect

75. A nurse is waiting for a report to be sent by fax. The machine activates but instead of the report, the nurse
received a sexually oriented photograph. The MOST appropriate nursing action is to:
a. call the nursing supervisor and report the incident
b. cut the photograph and throw it away.
c. call the police
d. call the unit who sent it and ask for the name of person who sent the photograph

76. Ms. F.X.. has been admitted with right upper quadrant pain and has been placed on a low fat diet. Which of the
following trays would be acceptable for her?
a. liver, fried potatoes and avocado
b. whole milk, rice and pastry
c. ham, mashed potatoes, cream peas
d. skim milk, lean fish, tapioca pudding

77. Negligence in the practice of nursing can be a ground for:


29
a. revocation of license by the Ombudsman
b. revocation of license by the BON
c. revocation of license by the DOH
d. revocation of license by the Nursing Department

78. One way of verifying that the right message/doctor’s order was communicated effectively is by:
a. phrasing intelligently
b. repeating the order message
c. documenting
d. speaking distinctly using enough volume

79. Kelly identified five categories of followers in an organization: sheep, yes people, alienated, survivor and effective
followers. If we choose to be successful nursing professionals, we have to become one these and influence other:
a. “YES” people c. effective followers
b. alienated d. sheep

80. A research on “Surgical Instrument Availability” will help the surgical team:
a. restructure OR scheduling system
b. streamline instrument inventory
c. improve instrument decontamination
d. ensure efficient completion of surgical instrument

81. Maria is administering a cleansing enema to a patient with fecal impaction. Before administering the enema, she
should place the patient in which of the following positions?
a. on the right side of the body with the head of the bed elevated 45 degrees
b. left Sim’s position
c. on the left side of the body with the head of the bed elevated 45 degree
d. right Sim’s position

82. The functionality and integrity of instruments and medical devices used in surgical procedure, is the responsibility
of the:
a. surgeon c. OR nurse
b. bio-med technician d. scrub nurse

83. Maria will be preparing a patient for thoracentesis. She should assist the patient to which of the following
positions for the procedure?
a. prone with the head turned to the side and supported by a pillow
b. lying in bed on the affected side with the head of the bed elevated 45 degrees
c. Sim’s position with the head of the bed flat
d. Lying in bed on the unaffected side with the head of the bed elevated 45 degrees

84. When a patient comes to the clinic for an eye examination, the ophthalmologists administers phenylephrine 2.5%
drops to:
a. dilate retinal blood vessels
b. anesthetize the cornea
c. dilate the pupil
d. removed any obstruction on the cornea

85. Which of the following statements by Ms. S.O. a chemotherapy patient with a low WBC count, a low platelet
count and a hemoglobin measurement of 5.6 g would indicate the need for further teaching?
a. “My lips are dry an d cracking. I need some lubricant”
b. “My husband and I have been using vaginal lubrication before my intercourse”
c. “I check my mouth and teeth after each meal”
d. “I’ve been very constipated and need an enema”

86. Patients undergoing surgery display different levels of anxiety. This is researchable. At present, it has been found
out that music can decreases anxiety. What other factors can reduce anxiety that is currently done among
postoperative cases?
30
a. pre-anesthetic drugs c. shower prior to surgery
b. pre-operative visit d. presence of any members of the family

87. According to AORN recommended practices, surgical attire intended only for use within the surgical suite should
be worn within the:
a. restricted semi-restricted and unrestricted areas
b. restricted area only
c. operating room only
d. restricted and semi-restricted areas

88. A heavily researched topic in infection control is about the single most important procedure for preventing
hospital-acquired infections. What is this procedure called?
a. handwashing c. use of facemask
b. use of scrub suite d. brain washing

89. Surgical instruments are expensive and are a lifetime investment of the O.R. To ensure quality of these
instruments, which criterion is evaluated?
a. shelf life c. cost
b. reusability d. integrity and functionality after each use and processing

90. Mr. Que, 61 years of age is suffering from rheumatoid arthritis. He asks if there is a cure for RA. The MOST
appropriate response is:
a. “No, most patients with RA also develop osteoarthritis.”
b. “Yes, but the patient must take medication for at least 10 years.”
c. “Yes, new drugs being developed offer a cure.”
d. “No, but new drugs being developed can interfere with the body’s reaction to inflammation
and better control the disease process.”

91. Rita is assigned to care for group of patients. On review of the patient’s medical record, she determines that
which patient is at risk for fluid volume deficit?
a. A client with CHF
b. A client receiving frequent wound irrigations
c. A client with colostomy
d. A client with decreased kidney function

92. In providing general information and assessing the patients level of interest or reaction to surgery, which of the
following nursing intervention will received priority?
a. tell the patient when the surgery is scheduled
b. let the patient know that the family will be kept in formed
c. explain details of perioperative preparation with a tour and viewing of area and equipments
d. offer general information about the surgery

93. In the event of a fire, OR staff are instructed to follow the international RACE. What does R stand for?
a. Rest b. Rescue c. respond d. run

94. In preparing the patient for endoscopic examinations of the upper GI tract, the patient’s pharynx is anesthesized
with Xylocaine spray. Nursing interventions for post-endoscopic examination include:
a. keeping patient NPO until gag reflex returns
b. prohibiting smoking before the test
c. withholding anticholinergic medications
d. allowing fluids up to 4 hours before examination

95. Modern medicine has divided the human body into different systems/organs or the so called “specialty medicine
practice”. Collaboration of the specialists will in effect treat the client as:
a. multiple specialty c. one whole human being
b. multi-organ d. compartmentalized body
31
96. Discharge teaching for the client with hypoparathyroidism should include which of the following instructions:
a. use OTC vitamin D preparation
b. avoid strenuous ex ercises
c. supplement calcium intake
d. avoid diuretics to minimize calcium loss

97. An important nursing intervention goal to establish for Mang Carlos who has iron-deficiency anemia is:
a. alternate periods of rest and activity to balance oxygen supply and demand
b. increase fluids to stimulate erythropoises
c. decrease fluids to prevent sickling of RBC’s
d. use birth control to avoid pregnancy

98. KN is monitoring the status of a postoperative patient. He would become MOST concerned with which of the
following signs which could indicate an evolving complication?
a. a negative Homan’s sign
b. BP of 110/170 mmHg and a pulse of 80 beats per minute
c. increasing restlessness
d. hypoactive bowel sounds in all four quadrants

99. The Patient’s Bill of rights helps practitioners provide more effective patient care. Which of the following should
NOT be included in the list?
a. be informed of administrative and policies and practices
b. considerate and respectful ca re
c. confidentiality of communications and records
d. relevant current and understandable information concerning diagnosis, treatment diagnosis, specific
procedures treatment and risks involved.

100. Disposal of medical records in government hospitals/institutions must be done in close coordination with what
agency?
a. DOH
b. Records Management and Archives Office (RMAO)
c. DILG
d. MMDA

-----------------------------------------------------------------------------------------------------------------------------------------------
-----

rioSET A Seat
No.____
---------------------------------------------------------------------------------------------------------------------------------
NURSING PRACTICE 3B:
NURSING CARE OF CLIENT WITH PHYSIOLOGICAL
AND PSYCHOSOCIAL ALTERATIONS
DIRECTION: Choose the letter of the BEST answer by shading the corresponding letter of your choice on the answer
sheet provided. STRICTLY NO ERASURE!

Situation 1: It is Cancer Consciousness Week and you are participating in an Early Cancer Detection Drive of the
Department of Health.

1. Based on the DOH and World Health Organization (WHO) guidelines, the mainstay for early detection method for
breast cancer that is recommended for developing countries is:

32
A. a monthly breast self examination (BSE) and an annual health worker breast examination
(HWBE)
B. an annual hormone receptor assay
C. an annual mammogram
D. a physician conduct a breast clinical examination every 2 years

2. The purpose of performing the breast self examination (BSE) regularly is to discover:
A. fibrocystic masses C. areas of thickness or fullness
B. cancerous lumps D. changes from previous BSE

3. If you are to instruct a postmenopausal woman about BSE, when would you tell her to do BSE:
A. on the same day of each month C. on the first day of her menstruation
B. right after the menstrual period D. on the last day of her menstruation

4. During breast self-examination, the purpose of standing in front of the mirror it to observe the breast for:
A. thickening of the tissue C. lumps in the breast tissue
B. axillary D. change in size and contour

5. When preparing to examine the left breast in a reclining position, the purpose of placing a small folded towel under
the client’s left shoulder is to:
A. bring the breast closer to the examiner’s right hand
B. tense the pectoral muscle
C. balance the breast tissue more evenly on the chest wall
D. facilitate lateral positioning of the breast

Situation 2: Ensuring safety is one of your most important responsibilities. You will need to provide instruction and
information to your clients to prevent complications.

6. LM has chest tube attached to a pleural drainage system. When caring for LM you should:
A. change the dressing daily using aseptic technique
B. empty the drainage system at the end of the shift
C. palpate the surrounding areas for crepitus
D. clamp the chest tube when suctioning

7. After pelvic surgery, the sign that would be indicative of a developing thrombophlebitis would be:
A. a tender, painful area on the leg C. a pitting edema of the ankle
B. pruritus on the calf and ankle D. a reddened area of the ankle

8. To prevent recurrent attacks on FT who has glomerulonephritis, you should instruct her to:
A. continue to take the same restrictions on fluid intake
B. seek early treatment for respiratory infections
C. avoid situations that involve physical activity
D. take showers instead of tub bath

9. GT had a laryngectomy. He is now for discharge. He verbalized his concern regarding his laryngectomy tube being
dislodged. What would you teach him FIRST?
A. reinsert another tubing immediately C. recognize that prompt closure of the tracheal opening
B. keep calm because there is no immediate D. notify the physician at once
emergency

10. When caring for TU after an exploratory chest surgery and pneumonectomy, your PRIORITY would be to
maintain:
A. chest tube drainage C. ventilation exchange
B. blood replacement D. supplementary oxygen

Situation 3: Severe burn is one of the most devastating kinds of injury one can experience. It can affect any group.
You have been ready to provide holistic care for patients with severe burns.

33
11. A burn that is white, painless, and leathery in texture describes a:
A. second degree burn C. deep partial thickness burns
B. third degree or full thickness burn D. first degree or superficial burns

12. Critically ill patients are at high risk for the following complication during the emergent phase:
A. myocardial infarction C. burn shock
B. neurogenic shock D. contractures

13. The MOST effective method of delivering pain medication during the emergent phase is:
A. intramuscularly C. orally
B. subcutaneously D. intravenously

14. Edema presents a significant problem in burn wounds because:


A. loss of protein prevents tissue repair
B. edema impedes tissue perfusion/oxygenation
C. edema provides a milieu for bacterial proliferation
D. edema can produce a tourniquet effect

15. Which of the following can be a fatal complication of upper airway burns?
A. stress ulcers C. shock
B. hemorrhage D. laryngeal spasms and swelling

Situation 4: You are assigned to take care of four patients with different conditions.

16. KJ, who is to have a kidney transplant asks you how long will he take azathioprime (Imuran), cyclosporine and
prednisone? You recognized that KJ understood the teaching when he states, “I must take these medications:
A. until the anastomosis heals C. until the supply is over
B. during the preoperative period D. for the rest of my life

17. After the kidney transplant, you must observe KJ for signs of rejection which includes:
A. fever and weight gain C. polyuria and jaundice
B. hematuria and seizure D. moon face and muscle atrophy

18. FB, 28 years old with chronic renal disease plans to receive a kidney transplant. Recently, FB was told by his
physician that he was a poor candidate for transplant because of his hypertension and diabetes mellitus. Now, FB tells
you “I want to go off dialysis, I’d rather not live than to be in this treatment the rest of my life”. How would you
respond to him?
A. leave the room and allow him to collect his thoughts
B. tell FB that “ We all have days when we don’t feel like going on”
C. tell FB that “ Treatments are only three times a week, you can live with that”
D. take a sit next to him and sit quietly

19. DS signed a consent form for participation in a clinical trial for implantable cardioverter defibrillators. Which
statement by DS indicates the need for further teaching before true informed consent can be obtained?
A. “a wire from the generator will be attached to my heart”
B. “the physician will make a small incision in my chest wall and place the generator there”
C. “I wonder if there is another way to protect these bad rhythms”
D. “this implanted defibrillator will protect me from those bad rhythms my heart goes into”

20. KP is participating in a cardiac study in which his physician is directly involved. Which statement by KP indicates a
lack of understanding about his rights as a research study participant?
A. “My confidentiality will not be compromised in this study”
B. “ I understand the risk associated in this study”
C. “I can withdraw from the study anytime”
D. “ I’ll have to find a new physician if I don’t complete this study”

Situation 5. You are assigned in the neurology stroke unit. To prepare for this assignment, you should be able to
answer the following questions.
34
21. Which of the following statements can BEST describe/define stroke or brain attack?
A. it occurs when circulation to a part of the brain is disrupted
B. it is usually caused by abuse of prescribed medications
C. it is caused by a cerebral hemorrhage
D. it may be the results of a transient ischemic attack (TIA)

22. Several diagnostic tests may be ordered for proper evaluation. The purpose of each of the following diagnostic
examination is correct EXCEPT:
A. Cerebral Angiography – is used to identify collateral blood circulation and may reveal site of rupture or occlusion
B. ECG – may reveal abnormal electrical activity, such as focal slowing and assess amount of brain wave
activity.
C. MRI – may reveal the site of infarction, hematoma and shift of brain structures
D. PET Scanning – may reveal information on cerebral metabolism and blood flow characteristics.

23. Which of the following is the MOST common cause of stroke or brain attack?
A. embolism C. cerebral arterial spasm
B. hemorrhage D. thrombosis

24. To guide you in your assessment, it is also important for you to remember that the clinical features of stroke vary
with the following factors EXCEPT:
A. severity of damage C. artery affected
B. gender D. the extent of collateral circulation

25. It is important for you to also teach clients and their families who are at risk to observed primary prevention
which includes the following EXCEPT:
A. maintain serum cholesterol level between 220 and 180 mm/dL
B. treat transient ischemic attacks (TIA) early
C. teach preventive health behaviors (consequences of smoking, obesity, alcoholism, drug abuse) to children
of patients with stroke
D. screen for systolic hypertension

Situation 6:Foot care among patients with peripheral vascular problems is very important.

26. When teaching a client with peripheral vascular disease about foot care, you should include which instructions:
A. avoid wearing canvas shoes C. avoid use of cornstarch on the foot
B. avoid using a nail clipper to cut toe nails D. avoid wearing cotton socks

27. FT, who has no known history of peripheral vascular disease, comes to the emergency room complaining of
sudden onset of lower leg pain. Inspection and palpation reveal absent pulses, paresthesia and a mottled, cyanotic,
cold, cadaverous left calf. While the physician determines the appropriate management, you should:
A. shave the affected leg in anticipation of surgery C. keep the affected leg level or slightly
dependent
B. place a healing pad around the calf D. elevate the affected calf as high as possible

28. Peripheral neuropathies primarily affect:


A. sensory functions C. optic functions
B. vascular functions D. motor functions

29. Peripheral neuropathy can BEST be controlled by:


A. good glucose control C. vitamin supplement
B. steroid therapy D. nothing, there is no slowing the process

30. In addition to clients with diabetes mellitus you must be aware that acute hypoglycemia can also develop in a
client with:
A. hypertension C. liver disease
B. hyperthyroidism D. diabetes insipidus

35
Situation 7: You are assigned to take care of a group of elderly patients. Pain and urinary incontinence are their
common concerns. You should be able to address their concerns in a holistic manner.

31. The WHO analgesic ladder provides the health professional with:
A. specific pain management choices based on severity of pain
B. general pain management choices based on level of pain
C. pharmacologic and nonpharmacologic pain management choices
D. nonpharmacologic interventions based on level of pain

32. As a nurse caring for patients in pain, you should evaluate for opioid side effects which include the following
EXCEPT:
A. pruritus C. constipation
B. respiratory depression D. physical dependence

33. Which of the following statements about cancer pain is NOT true?
A. opioids are drugs of choice for severe pain
B. pain associated with cancer and the terminal phase of the disease occurs in majority of patients
C. under treatment of pain is often due to a clinician’s failure or inability to evaluate or appreciate the severity
of the client’s problem
D. adjuvant medications such as steroids, anti convulsants, nonsteroidal anti-inflammatory
drugs enhance pain perception

34. TR has been on morphine on a regular basis for several weeks. He is now complaining that the usual dose he has
been receiving is no longer relieving his pain as effectively. Assuming that nothing has changed in his condition, you
would suspect that TR is:
A. becoming psychologically dependent C. needing to have the morphine discontinued
B. developing tolerance to the morphine D. exaggerating his level of pain

35. The guidelines for choosing appropriate nonpharmacologic intervention for pain include all of the following
EXPECT:
A. effectiveness for patient C. skill of the clinician health professional
B. pain problem identification D. type of opioid being used

Situation 8: To be able to provide care for patients in the critical areas, you should look into factors that will
enhance your ability to provide quality nursing care.

36. Research study show that nurses who work with critically patients as opposed to nurses who work with less acute
patient:
A. are more satisfied with their role C. are most acceptable to burn out
B. move a greater support system D. experience greater stress

37. Which of the components of HARDINESS has been linked to burnout?


A. less commitment to work C. a sense of control over the patient
B. perception of change D. sense of control to life

38. Nurses who work with critically ill patients should base their practice on all of the following EXCEPT:
A. recognition and appreciation of a person’s unique and social environmental relationships
B. delegated responsibility
C. thorough knowledge of the interrelatedness of body system
D. appreciation of the collaborative role of all health team members
39. Common aspects of the critical care nursing role include:
A. disaster management C. direct care provider
B. staff liaison D. community referral

40. Which of the following interventions would support your patient’s circadian rhythm cycle?
A. putting a wall clock up on your patient’s room
B. decreasing environmental noise
C. encouraging normal bowel movement
36
D. dimming light during normal sleeping time

Situation 9: To ensure continuity of care and for legal purposes, you have important responsibilities to accurately
document all nursing activities.

41. For the past 24 hours, TD with dry skin and dry mucous membranes has had a urine output of 600 m and a fluid
intake of 800 ml. TD’s urine is dark amber. These assessments indicate which nursing diagnosis?
A. Impaired urinary elimination C. Excessive fluid volume
B. Deficient fluid volume D. Imbalanced nutrition: less than body requirement

42. Which document addresses the patient’s right to information, informed consent and treatment refusal?
A. Code for Nurses C. Patient’s Bill of Rights
B. Nursing Practice Act D. Standard of Nursing Practice

43. You are caring for GG with a history of falls. The FIRST PRIORITY when caring for GG who is at risk for falls is:
A. instruct GG not to get out of bed unassisted
B. keep the bedpan available so she does not have to get out of bed
C. placing the call light for easy access
D. keep the bed at the lowest position ever

44. Shortly after being admitted to the CCU for acute MI, JJ reports midsternal chest pain radiating down the left arm.
You notice that JJ is restless, slightly diaphoretic, and has a temperature of 37.8 deg C, heart rate of 10 beats/min.;
regular, slightly labored respirations at 26 breaths/min and a blood pressure of 150/90 mmHg. Which nursing
diagnosis takes HIGHEST PRIORITY?
A. decreased cardiac output C. acute pain
B. anxiety D. risk for imbalanced body temperature

45. FF, has a nursing diagnosis of “Risk for injury related to adverse effects of potassium-wasting diuretics”. What’s
the correct written client outcome for this diagnosis?
A. FF states the importance of eating potassium rich foods daily
B. Upon discharge, FF knows which food sources are rich in potassium
C. Upon discharge, FF correctly identifies three potassium rich foods
D. FF knows all the complications of the disease process

Situation 10: You are taking care of LC who develops acute respiratory distress. An endotracheal tube had to be
inserted to correct the hypoxia.

46. The primary purpose of the endotracheal tube cuff is to:


A. seal off the oropharynx from the nasopharynx C. seal off the oropharynx from the esophagus
B. seal off the lower airway from the esophagus D. seal off the lower airway from the upper airway

47. Endotracheal tube size indicated on the tube reflects what measurements:
A. the circumference size of the tube C. the internal diameter of the tube
B. the length of the tube D. the length of the person’s airway

48. In adults, an inflated E-T tube cuff is necessary for mechanical ventilation primarily because:
A. it seals off the lower airway from the upper airway
B. it prevents air from getting into the stomach
C. it seals off the nasopharynx from the oropharynx
D. it prevents stomach contents from getting into the lungs

49. Endotracheal tube size indicated on the tube reflects what measurements:
A. the internal diameter of the tube C. the circumference size of the tube
B. the length of the person’s airway D. the length of the tube

50. Which of the following statements is TRUE about securing the artificial airway?
A. artificial airways must be secured directly to the patient
B. the airway is generally sutured in place
37
C. a nasotracheal tube does not require securing
D. the inflated cuff provides sufficient securing

Situation 11: Because of the serious effects of severe burns, management requires a multidisciplinary approach.
You have important responsibilities as a nurse.

51. When caring for DS, who sustained 40% severe flame burn yesterday, which among these interventions should
be your PRIORITY?
A. provide a calm, efficient and safe environment
B. keep the body parts in good alignment to prevent contractures
C. assess for airway, breathing and circulation problems
D. assess the injury for signs of sepsis

52. Your primary therapeutic goal for DS during the ACUTE PHASE is:
a. wound healing c. emotional support
b. reconstructive surgery d. fluid resuscitation

53. CV who sustained upper torso and neck burns. Which action is MOST likely to cause a functional contracture?
a. hourly hyperextension neck exercises
b. helping the patient to a position of comfort
c. encouraging self-care
d. discouraging pillows behind the head

54. AW, 3 year old boy just sustained full thickness burns of the face, chest and neck. What will be your PRIORITY
nursing action?
a. Risk for infection related to epidermal disruption
b. Impaired urinary elimination related to fluid loss
c. Ineffective airway clearance related to edema
d. Impaired body image related to physical appearance

55. FG, with a full thickness burns involving entire circumference of an extremity will require frequent peripheral
vascular checks to detect:
a. hypothermia c. arteriosclerotic changes
b. ischemia d. adequate wound healing

Situation 12: Infection can cause debilitating consequences when host’s resistance is compromised and
environmental factors are favorable. As a nurse you have important roles and responsibilities in infection control.

56. EF was admitted to the hospital with a tentative diagnosis of acute pyelonephritis. To assess her risk factors,
what question should you ask?
a. “Have you taken any analgesic recently?”
b. “Do you have pain at your back?”
c. “Do you hold your urine for a long time before voiding?”
d. “Have you had any sore throat lately?”

57. While caring for a patient with an infected surgical incision, you observe for signs of systemic response. These
include all of the following EXCEPT:
a. a febrile state due to release of pyrogens
b. anorexia, malaise, and weakness
c. loss of appetite and pain
d. leukopenia due to increased WBC production

58. One of the MOST effective nursing procedures for reducing nosocomial infection is:
a. proper handwashing technique
b. aseptic wound care
c. control of upper respiratory tract infection
d. administration of prophylactic antibiotic
38
59. A wound that has hemorrhaged has increased risk for infection because:
a. dead space and dead cells provide a culture medium
b. retrograde bacterial contamination may occur
c. the tissue becomes less resilient
d. of reduced amounts of oxygen and nutrients are available

60. You are instructing EP regarding skin tests for hypersensitivity reactions. You should teach her to:
a. stay out of the sun until the skin tests are read
b. come back on the specified date to have the skin tests read
c. wash skin test areas with soap and water daily
d. keep skin test areas moist with mild lotion.

Situation 13: TR attends a Health Education Class on colostomy care. The following are taken up: types of
ostomies, indications and care.
61. A colostomy can BEST be defined as:
a. cutting the colon and bringing the proximal end through the abdominal wall
b. creating a stomal orifice from the ileum
c. excising a section of the colon and doing an end-to-end anastomosis
d. removing the rectum and suturing the colon to the anus.

62. When an abdominoperineal resection is done, the patient should be informed he/she will have a;
a. temporary colostomy c. transverse loop colostomy
b. permanent colostomy d. double-barreled

63. A colostomy patient who wishes to avoid flatulence should not eat the following EXCEPT:
a. corn and peanuts c. mangoes and pineapples
b. cabbage and asparagus d. chewing gum and carbonated beverages

64. During the first post operative week, the nurse can BEST help the patient with a colostomy to accept the change
in body image by:
a. changing the dressing just prior to meals
b. encouraging the patient to observe the stoma and its care
c. deodorizing the room periodically with a spray can
d. applying a large bulky dressing over the stoma to decrease odors

Situation 14: These are gastrointestinal disease that can compromise life and that would necessitate extensive
surgical management. You are assigned to take care of a patient with such a condition.

66. BC diagnosed with cancer of the sigmoid colon is to have an abdominoperineal resection with a permanent
colostomy. Before surgery, a low residue diet is ordered. You explain to BC that this is necessary to:
a. prevent irritation of the intestinal mucosa
b. reduce the amount of stool in the large bowel
c. limit production of flatus in the intestines
d. lower the bacterial count in the GI tract

67. Several days prior to bowel surgery, the patient may be given sulfasuxidine and neomycin, primarily to:
a. soften the stool by retaining water in the colon
b. reduce the bacterial content of the colon
c. empty the bowel of solid waste
d. promote rest of the bowel by minimizing peristalsis

68. To promote perineal wound healing after an abdominoperineal resection, you should encourage BC to assume:
a. dorsal recumbent position
b. left or right Sim’s position
c. left or right side lying position
d. knee-chest position

39
69. BC returns from surgery with a permanent colostomy. During the 24 hours, the colostomy does not drain. You, as
the nurse should realize that this is a result of:
a. the absence of intestinal motility
b. a presurgical decrease in fluid intake
c. proper functioning of the nasogastric tube
d. intestinal edema following surgery

70. On the second day following abdominoperineal resection, you anticipate that the colostomy stoma will appear:
a. moist, pink, with flushed skin and painful when touched
b. moist, red and raised above the skin surface
c. dry, pale pink and with flushed skin
d. dry, purple and depressed below the skin surface

Situation 15: Specific surgical interventions may be done when lung cancer is detected early. You have important
peri-operative responsibilities in caring for patients with lung cancer.

71. GM is scheduled to have lobectomy. The purpose of closed chest drainage following a lobectomy is:
a. expansion of the remaining lung
b. facilitation of coughing
c. prevention of mediastinal shift
d. promotion of wound healing

72. Following thoracic surgery, you can BEST help GM to reduce pian during the deep breathing and coughing
exercises by:
a. splinting the patient’s chest with both hands during the exercises
b. administering the prescribed analgesic immediately prior to exercises
c. providing rest for 6 hours before exercises
d. placing the patient on his/her operative side during exercises

73. During the immediate post operative period following a pneumonectomy, deep tracheal suction should be done
with extreme caution because:
a. the remaining normal lung needs minimal stimulation
b. the patient will not be able to tolerate coughing
c. the tracheobronchial tree are dry
d. the bronchial suture line maybe traumatized

74. What should you do as a nurse when the chest tubing is accidentally disconnected?
a. reconnect the tube c. notify the physician
b. change the tubing d. clamp the tubing

75. Which of the following observations indicates that the closed chest drainage system is functioning properly?
a. less than 25 ml drainage in the drainage bottle
b. absence of bubbling in the suction-control bottle
c. the fluctuating movement of fluid in the long tube of the water-seal bottle during inspiration
d. intermittent bubbling through the long tube of the suction control bottle.

Situation 16: Renal stones can cause one of the most excruciating pain experienced by a patient. As a nurse of BL
which of the following nursing diagnosis will be your PRIORITY?

76. BL was brought to the Emergency Room for severe left flunk pain, nausea and vomiting. The physician gave a
tentative diagnosis of right ureterolithiasis. As the nurse of BL which of the following nursing diagnosis will be your
PRIORITY?
a. imbalance nutrition: less than body requirements
b. impaired urinary elimination
c. acute pain
d. risk for infection

77. Which of the following is the appropriate intervention for BL who has ureterolithiasis?
40
a. inserting an indwelling urinary catheter
b. administering opioid analgesics preferably intravenously
c. administering intravenous solution at a keep vein open rate
d. inserting a nasogastric tube (low suction)

78. You are caring for YA, 30 year old business woman, with renal stones. Her skin and mucous membranes are dry
and her 24 hour intake and output record reveal an oral intake of 900 ml and a urinary output of 700 ml. Her urine
is dark amber. Based on the above data, your nursing diagnosis is:
a. imbalance nutrition, less than body requirements
b. fluid volume deficit
c. impaired urinary elimination
d. knowledge deficit regarding health

79. KJ has an indwelling urinary catheter and she is suspected of having urinary infection. How should you collect a
urine specimen for culture and sensitivity?
a. clump tubing for 60 minutes and insert a sterile needle into the tubing above the clamp to aspirate urine
b. drain urine from the drainage bag into the sterile container
c. disconnect the tubing from the urinary catheter and let urine flow into a sterile container
d. wipe the self-sealing aspiration port with antiseptic solution and aspirate urine with a sterile
needle

80. You are caring for WE, a 56 year old man who is dehydrated and with urinary incontinent. Upon physical
examination, you noted perineal excoriation. What will be your PRIORITY intervention?
a. orient him to time, person and place
b. offer the bed pan every 4 hours
c. encourage oral fluid intake
d. keep the perineal area clean, and dry

Situation 17: You are caring for several patients with various disease problems.

81. You are obtaining a history of MR. who is admitted with acute chest pain. Which question will be MOST HELPFUL
for you to ask?
a. Why do you think you had a heart attack?
b. Do you need anything now?
c. What seem you doing when the pain started?
d. Has anyone in your family been sick lately?

82. BO who received general anesthesia returns from surgery. Post-operatively, which nursing diagnosis takes
HIGHEST PRIORITY for BO?
A. impaired physical mobility related to surgery
B. decrease fluid volume related to blood and fluid loss from surgery
C. risk for infection related to anesthesia
D. acute pain related to surgery

83. WW is blind. She is admitted for treatment of gastroenteritis. Which nursing diagnosis takes HIGHEST PRIORITY
for WW?
A. anxiety C. activity intolerance
B. risk for injury D. impaired physical mobility

84. You are documenting your care for CC who has iron deficiency anemia. Which nursing diagnosis is MOST
appropriate?
A. ineffective breathing pattern C. deficient fluid volume
B. impaired gas exchange D. ineffective airway clearance

85. RR, age 89, has terminal cancer, he demonstrates signs of dementia. You should give HIGHEST PRIORITY to
which nursing diagnosis:
A. risk for injury C. ineffective cerebral tissue perfusion
B. bathing or hygiene self care deficit D. dysfunctional grieving
41
Situation 18: The physician has ordered 3 units of whole blood to be transfused to WQ following a repair of a
dissecting aneurysm of the aorta.

86. You are preparing the first unit of whole blood for transfusion. From the time you obtain it from the blood bank,
how long should you infuse it?
A. 6 hours C. 4 hours
B. 1 hour D. 2 hours

87. What should you do FIRST before you administer blood transfusion?
A. verify client identity and blood product, serial number, blood type, cross matching results, expiration
date
B. verify client identity and blood product serial number, blood type, cross matching results, expiration
date with
another nurse
C. check IV site and use appropriate BT set and needle
D. verify physician’s order

88. As WQ’s nurse, what will you do AFTER the transfusion has started?
A. add the total amount of blood to be transfused to the intake and output
B. discontinue the primary IV of Dextrose 5% Water
C. check the vital signs every 15 minutes
D. stay with WQ for 15 minutes to note for any possible BT reactions

89. WQ is undergoing blood transfusions of the first unit. The EARLIEST signs of transfusion reactions are:
A. oliguria and jaundice C. hypertension and flushing
B. urticaria and wheezing D. headache, chills, fever

90. In case WQ will experience an acute hemolytic reaction, what will be your PRIORITY intervention?
A. immediately stop the blood transfusion, infuse Dextrose 5% in Water and call the physician
B. stop the blood transfusion and monitor the patient closely
C. immediately stop the BT, infuse NSS, call the physician, notify the blood bank
D. immediately stop the BT, notify the blood bank and administer antihistamines

Situation 19. The kidneys have very important excretory, metabolic, erythropoietic functions. Any disruptions in the
kidney’s functions can cause disease. As a nurse it is important that you understand the rationale behind the
treatment regimen used.

91. PL, who is in acute renal failure, is admitted to the Nephrology Unit. The period of oliguria usually lasts for about
10 days. Which assessment parameter for kidney function will you use during the oliguric phase?
A. urine output directly related to the amount of IV fluid infused
B. urine output is less than 400 ml/24 hours
C. urine output of 30-60 ml/hour
D. no urine output, kidneys in a state of suspension

92. During the shock phase, what is the effect of the rennin-aldosterone-angiotensin system on renal function?
A. increased urine output, increased absorption of sodium and water
B. decreased urine output, decreased absorption of sodium and water
C. increased urine output, decreased absorption of sodium and water
D. decreased urine output, increased absorption of sodium and water

93. As you are caring for PL who has acute renal failure, one of the collaborative interventions you are expected to do
is to start hypertonic glucose with insulin infusion and sodium bicarbonate to treat:
A. hyperkalemia C. hypokalemia
B. hypercalcemia D. hypernatremia

94. BN, 40 year old with chronic renal failure. An arteriovenous fistula was created for hemodialysis in his left arm.
What diet instructions will you need to reinforce prior to his discharge?
42
A. drink plenty of water C. monitor your fruit intake and eat plenty of
bananas
B. restrict your salt intake D. be sure to eat meat every meal

95. BN, is also advised not to use salt substitute in the diet because:
A. salt substitute contain potassium which must be limited to prevent arrhythmias
B. limiting salt substitutes in the diet prevents a buildup of waste products in the blood
C. fluid retention is enhanced when salt substitutes are included in the diet
D. a substance in the salt substitute interferes with fluid transfer across the capillary membrane

Situation 20. You are assigned to take care of a group of elderly patients. Pain and urinary incontinence are common
concerns experienced by them. You should be able to address the concerns in a holistic manner.

96. Pain in the elder persons require careful assessment because they:
A. experienced reduce sensory perception
B. have increased sensory perception
C. are expected to experience chronic pain
D. have a decreased pain threshold

97. Administration of analgesics to the older persons requires careful patient assessment because older people:
A. are more sensitive to drugs
B. have increased hepatic, renal and gastrointestinal function
C. have increased sensory perception
D. mobilize drugs more rapidly

98. The elderly patient is at higher risk for urinary incontinence because of:
A. increased glomerular filtration C. decreased bladder capacity
B. diuretic use D. dilated urethra

99. Which of the following is the MOST COMMON sign of infection among the elderly?
A. decreased breath sounds with crackles C. pain
B. fever D. change in mental status

100. Priorities when caring for the elderly trauma patient:


A. circulation, airway, breathing C. airway, breathing, disability (neurologic)
B. disability (neurologic), airway, breathing D. airway, breathing, circulation

SET A Seat No.____


---------------------------------------------------------------------------------------------------------------------------------
NURSING PRACTICE 3C:
NURSING CARE OF CLIENT WITH PHYSIOLOGICAL
AND PSYCHOSOCIAL ALTERATIONS
DIRECTION: Choose the letter of the BEST answer by shading the corresponding letter of your choice on the answer
sheet provided. STRICTLY NO ERASURE!

Situation 1: The nurse is envisioning a career path in mental health psychiatric nursing. As a beginning professional
nurse, she is guided with basic beliefs about the practice.

1. Which of the following statement reflects the scope of mental health psychiatric nursing?
A. it includes nursing actions aimed at returning the patient to his highest potential of productivity.
B. it is an integral aspect of all nursing and a specialty service to all people affected by mental
illness.
C. it includes nursing actions to reduce the rate of new cases of mental disorder in population

43
D. it consist of early recognition and treatment of mental disorders to reduce severity and duration of mental
illness

2. The beginning professional nurse can do mental health counseling with the following clients, EXCEPT:
A. actively psychotic patients C. parents with child rearing concerns
B. out of school adolescents D. school children with behavioral problem

3. A professional responsibility of the mental health psychiatric nurse is to provide a safe and therapeutic environment.
This is BEST reflected in:
A. restraining patients who violates policies and do not follow schedule of activities.
B. maintaining a closed door policy to prevent patients from absconding.
C. keeping a restrictive environment to prevent patients from becoming assaultive and hostile
D. ensuring physical safety and maintaining therapeutic attitude towards the patients

4. The foundation of the therapeutic process is the therapeutic relationship. What is the essential component that the
nurse must bring to the relationship?
A. humor C. reframing
B. empathy D. confrontation

5. Which of these people, the highest in population groups that would need priority mental health therapy?
A. adults going through active skills C. young professionals entering the workplace
B. single elderly with no social support D. women preparing for overseas employment

Situation 2: Some activity therapies are organized and conducted in groups where nurses may participate in.

6. A mental health nurse may not be a member of this self help group because help given to members comes from
members themselves:
A. Remotivation Group C. Activity Therapy Group
B. Alcoholics Anonymous D. Art Therapy Group

7. Which of the following determines the success of client government groups?


A. willingness of psychiatric professionals to be open and receptive to client’s ideas and suggestions
B. a way of permitting clients provide themselves with a more creative and wholesome life
C. means to acquire a variety of social skills
D. opportunity to learn democratic living

8. Which of this client situation appropriately illustrate horticulture therapy?


A. the therapist brings bongos, tambourine, and bells and encourages client participation
B. Tommy, Karen, Jon and Pia play scrabble every night after supper
C. every afternoon, Vic goes in the garden where he work with plants, seedlings, tree planting
And watering them
D. Paul finds sketching relaxing and rewarding
9. Eva is a member of a group who controls endless talking. Her role is that of a/an:
A. blocker C. recognition seeker
B. self-confessor D. monopolizer

10. Community meetings are held as a part of milieu therapy on an in-patient psychiatric unit. The purpose of these
meetings would be:
A. to focus on issue arising from group living
B. to encourage expression on topics of interest
C. to provide direction from the treatment plan
D. to encourage expression of intrapsychic conflicts

Situation 3: The patient who is depressed will undergo electroconvulsive therapy.

11. Studies on biological depression support electroconvulsive therapy as a mode of treatment. The rationale is:
A. ECT produces massive brain damage which destroys the specific area containing memories related to
the events surrounding the development of psychotic condition
44
B. The treatment serves as a symbolic punishment for the client who feels guilty and worthless
C. ECT relieves depression psychologically by increasing the norepinephrine level
D. ECT is seen as a life-threatening experience and depressed patients mobilize all their bodily defenses
to deal with this attack.

12. The preparation of a patient for ECT ideally is MOST similar to preparation for a patient for:
A. electroencephalogram C. general anesthesia
B. X-ray D. electrocardiogram

13. Which of the following is a possible side effect which you will discuss with the patient?
A. hemorrhage within the brain C. encephalitis
B. robot-like body stiffness D. confusion, disorientation and short term
memory loss

14. Informed consent is necessary for the treatment for involuntary clients. When this cannot be obtained, permission
may be taken from the:
A. social worker C. next of kin or guardian
B. doctor D. chief nurse

15. After ECT, the nurse should do this action before giving the client fluids, food or medication:
A. assess the gag reflex C. next of kin or guardian
B. assess the sensorium D. check O2 Sat with a pulse oximeter

Situation 4: The community health nurse encounters special children in the community.

16. An individual with antisocial personality disorder lacks remorse, shame and guilt in going against the norms of society.
Psychodynamically, this defect in the personality reflects a disturbance of the:
A. ego C. ego ideal
B. super ego D. id

17. The nurse teaches parents about children’s beginning concepts of right and wrong by emphasizing child rearing
attitude and practices during the:
A. school age C. infancy period
B. toddler age D. latency period

18. It is BEST for parents to teach healthy interpersonal relationships to their children by:
A. modeling to their children
B. encouraging their children to attend secondary school
C. encouraging their children at home to behave properly
D. teaching their children good manners and right conduct

19. An important principle for the nurse to follow in interacting with retarded children is:
A. seen that if the child appears contented, his needs are being met
B. provide an environment appropriate to their development task as scheduled
C. treat the child according to his chronological age
D. treat the child according to his developmental level

20. Mental retardation is:


A. a delay in normal growth and development caused by an inadequate environment
B. a lack of development of sensory abilities
C. a condition of subaverage intellectual functioning that originates during the developmental
period and is associated with impairment in adaptive behavior
D. a severe lag in neuromuscular development and motor abilities

Situation 5: The nurse recognizes the need to learn to cope with stress and change. She becomes interested to practice
natural ways to enhance well being.

45
21. Lifestyle modification begins with:
A. minimizing eating in fast food restaurants
B. having an exercise regimen to follow regularly
C. recognizing the impact of unhealthy habits
D. avoiding pollutants in the environment

22. All of these are the behavior intervention to stress management. EXCEPT:
A. guided imagery C. meditation
B. pharmacotherapy D. progressive muscle relaxation

23. Basic to progressive muscle relaxation is:


A. focusing on an image to relax C. use of industrial equipment
B. relaxing muscles from tension D. stopping disturbing thoughts

24. Dietary practices are very important to the health of the Filipino family. The nurse needs to assess this
lifestyle because:
A. the nurse wants to change the eating patterns of the Filipino family
B. the nurse knows that being overweight is a major health hazard
C. the nurse wants to stop all the mainstream weight-loss diets
D. the nurse has to find out what people are eating

25. A young overweight adult smokes 5-10 sticks of cigarettes/day, ambitious, looks at life as challenging and perfect and
never considers change in his lifestyle, initially needs:
A. commitment C. skills to attempt change
B. information D. motivation

Situation 6: Bernie and John in their late 40’s have been married for 20 years and at the peak of their careers.
Suddenly, Bernie discovered that her husband was falling in love with another woman. Shaken by this situation, she
started to have problems sleeping and could not function well at work and at the risk of losing her job. John asked
forgiveness and regret very much the hurt his wife was going through and suffered guilt feelings:

26. Bernie and John are going through a:


A. situational crisis C. anticipated crisis
B. developmental crisis D. both developmental and situational crisis

27. All of these are characteristics of crisis EXCEPT:


A. a hazardous or threatening event occurs
B. it has a growth promoting potential
C. usual problem solving methods and coping mechanisms produce a solution
D. anxiety or depression continue to increase

28. The nurse employs this approach in crisis intervention:


A. problem-solving C. role-playing
B. behavior modification D. nurse-patient relationship

29. Assessment data of the nurse include all the following EXCEPT:
A. coping mechanisms C. perception of the event
B. situational support D. repressed problems

30. The duration of crisis usually lasts several days and usually:
A. 2 – 4 weeks C. 1 – 2 months
B. 1 – 2 weeks D. 4 – 6 weeks

Situation 7: Felisa has a ritualistic pattern of constantly washing her hands with soap and water followed by rubbing
alcohol.

31. This behavior is categorized as:


A. delusional C. neurotic
46
B. normal D. psychotic

32. A therapeutic intervention in this situation is:


A. avoid limits on her behavior to release her anxiety
B. call attention to her ritualistic pattern
C. provide alternative behaviors to deal with increased anxiety
D. ignore her behavior totally

33. The anxiety of Felisa is disabling and interferes with her job performance, interpersonal relationships and other
activities of daily living. To minimize such problems, she is likely to be given:
A. diazepam ( Valium ) C. imipramine Hcl ( Tofranil )
B. haloperidol ( Haldol ) D. chlorpromazine ( Thorazine )

34. Felisa understands the effects of her medicine when expresses:


A. “I should watch out for signs of sore lips or sore throat”
B. “I might have constipation”
C. “I might feel changes in my body temperature”
D. “ I should not drive or operate machines”

35. The level of anxiety that Felisa is experiencing is:


A. Panic C. Mild
B. Severe D. Moderate

Situation 8:As a professional, it is imperative that the nurse is accountable to oneself hence the importance of personal
and professional development.

36. Nurse: “ I feel personally involved with my client’s problems” demonstrates:


A. counter transference C. transference
B. empathy D. sympathy

37. The nurse has achieved self-awareness in which of the following verbalizations?
A. every time people around me yell, I feel upset and withdrawn
B. when the patient yelled at me I became speechless
C. with the patients tone of voice and stare, I got reminded of how my father would be so angry and
this made me anxious
D. I thought it was rude for the patient to yell hence I kept quiet

38. An accepting attitude requires being:


A. aware of ones biases C. non judgmental
B. tolerant of the faults of others D. in control of tendency to blame

39. Self-awareness, knowledge and understanding of human behavior and communication skills define what is essential in
caring for every nurse to be able to demonstrate:
A. positive self-projection C. therapeutic use of self
B. assertiveness D. self-mastery

40. Considering that man is by nature social, it is BEST for the nurse to gain self-awareness by:
A. participating in intensive group experiences
B. individual psychotherapy
C. hypnotherapy
D. writing an autobiography for self introspection

Situation 9: Recording and the nurse.

41. To facilitate identification of persons and relationships, the family nurse utilizes this diagrammatic representation of
members of a family and their relationships:
A. flowchart C. algorithm
B. genogram D. kardex
47
42. A problem oriented recording system that utilizes the problem solving process as the basis of patient care
management:
A. NANDA Diagnosis C. Gordon’s Functional Health Patterns
B. Nursing Care Plan D. Problem Oriented Medical Recording

43. This is a SOAP recording of the patient’s problem of “Nervousness”. Which is the subjective data?
A. Mr. Z was nervous during the interview, he moved frequently in the bed and his palms were sweaty.
B. Mr. Z does not seem to tolerate stress too well which will aggravate his cardiac condition. He understands
Little about his health which may be increasing his state of anxiety.
C. “I am nervous at times”. Exerts himself physically and is hesitant to discuss problems.
D. Mr. Z should:
1. demonstrate an ability to cope with nervousness
2. demonstrate an understanding of the relationship between his nervousness and cardiac condition

44. The nurse researcher mutually agreed with the research subjects that all personal data of the subjects shall be kept
confidential. To safeguard anonymity of data after the report is written, the nurse researcher should:
A. throw the papers in a wastebasket
B. keep the papers for 5 years and dispose of these after
C. burnt or shred the papers
D. have the papers recycle but not as writing scratch paper

45. A step by step procedure for the management of common problems is a :


A. Gavin chart C. decision tree protocol
B. pedigree chart D. problem listing

Situation 10:The nurse works with Mina to help her work through termination of the nurse-patient relationship.

46. Preparation for termination of the nurse-patient relationship begins during the:
A. termination phase C. pre-orientation phase
B. working phase D. orientation phase

47. Mina’s past reactions to ending relationships is withdrawal. The nurse assists her to practice better ways of coping
termination by providing opportunities to:
A. test new patterns of behavior C. conceptualize her problem
B. plan for alternatives D. value and find meaning in experience

48. During the early part of the interaction, the nurse asked after a period of silence. “Perhaps we would talk about my
leaving”. The nurse utilized which communication technique:
A. encouraging C. focusing on client
B. suggesting D. understanding

49. Mina: (angrily) “ I am angry, I should never have gotten involved with you” (silence)
Nurse: “You have reason to be disappointed and to feel angry. Just when we were getting somewhere.
I leave you.”
A. reflecting words C. orienting
B. understanding D. reflecting feelings

50. The objection of the nurse-patient relationship is to provide an opportunity of the patient to:
A. clarify problems C. have a corrective emotional experience
B. develop insights D. develop interpersonal relationship

Situation 11: As a program manager, the mental health psychiatric nurse is tasked to provide general patient
management.

51. The nurse is aware that identifying the aspects of general patient management and identifying interventions for
meeting these basic needs are distinctions of:
A. psychotherapy C. pharmacotherapy
48
B. therapeutic milieu D. behavior therapy

52. In order to get active participation of the clients to carry out the objective of the program, it is BEST for the nurse to
conduct a/an:
A. community meeting C. observation
B. survey D. selective interview of patients

53. Through the nurse’s role modeling of effective communication, the clients learned new ways of dealing with authority
figures. This gives the clients a venue to:
A. communicate C. identify their problems
B. socialize D. test new patterns of behavior

54. This element of communication facilitates evaluation of the program:


A. receiver C. sender
B. message D. feedback

55. The nurse’s style of leadership in milieu therapy is:


A. autocratic C. democratic
B. laissez-faire D. benevolent

Situation 12: Loretta is a 28 years old, unemployed patient, admitted to the psychiatric unit with a diagnosis of chronic
undifferentiated schizophrenia. She described herself as the “Virgin Mary” and her mission is to propagate peace. She was
observed laughing and talking to herself. Her thought processes were profoundly disorganized. She was also fearful and
suspicious of others.

56. Loretta’s claim of being the “Virgin Mary” is a/an:


A. imagination C. hallucination
B. delusion D. obsession

57. Her mission to propagate peace is:


A. a serious call for a need of transformation C. realistic and laudable
B. an attempt to overcome low self-esteem D. an advocacy that she can participate in

58. Which of this intervention would NOT be therapeutic in decreasing Loretta’s anxiety?
A. joke about her thought to help her feel at ease
B. listen to her thoughts and feelings
C. simply accept
D. do not convince her that her perception is unreal

59. The nurse evaluates that Loretta’s ready for a rehabilitation program when she:
A. approaches the nurse at frequent intervals C. goes to the coffee shop alone more often
B. ceases to talk about “Virgin Mary” D. carries a book “The Purpose Driven Life”

60. The nurse is leading a group meeting of patients to prepare them to be discharged. An appropriate goal for the group
members is to develop:
A. attitudes of society towards the mentally ill
B. skills for maintaining daily living
C. awareness of interpersonal patterns of interactions
D. insight into personal problems

Situation 13: A group of adult chronic schizophrenic patients were recommended to undergo social skills training.

61. The following are the objectives of a social skills training program EXCEPT:
A. explore deep seated intrapsychic conflicts C. help build self esteem and self confidence
B. practice skills in relating with people D. develop and practice general coping skills

62. Social skills training is NOT primarily indicated for psychiatric patients who are:
A. in acute stage of illness
49
B. having difficulties starting and maintaining interpersonal relationships
C. having chronic episodes of stress and anxiety while interacting with others
D. experiencing recurrence of symptoms in front of particular people or among people in general

63. The focus of the group interaction is “here and now”. An appropriate topic would be:
A. ways to celebrate Valentine’s Day in February
B. how to spend the summer vacation
C. an unforgettable experience as a child
D. how to tell a joke

64. An appropriate technique for the participants to practice how to communicate effectively is through/a:
A. lecture C. role play
B. seminar D. psychodrama

65. Considering that it is BEST to learn by example, it is MOST practical to:


A. model good social skills throughout the session
B. relate successful past experiences
C. invite a resource person
D. watch a movie

Situation 14: Community health nurses integrate their knowledge of mental health with their clients in their practice
when they do mental health counseling.

66. Who among these clients need immediate referral for psychotherapy?
A. those who are plotting to commit suicide
B. those who are not responding to usual motivators
C. those who are not solving their problems which they have the resources to solve
D. those who are engaging in self-defeating behaviors

67. All of these describe what counseling, EXCEPT:


A. helping people manage their own problems
B. assisting people to use their own resources
C. a set of technique, skills and attitudes
D. giving advices

68. This quality of the nurse allows “connecting with others feelings”:
A. warmth C. sensitivity
B. courage D. prudence

69. “ To refuse gossip” is a reflection of the mental health nurses quality of:
A. firmness C. judgment
B. coolness D. trustworthiness

70. Which of the following concepts BEST describe the nurse’s interaction when she/he can recognize and identify
feelings and emotions of another person without even having personally experienced those feelings and emotions?
A. over involvement C. transference
B. sympathy D. empathy

Situation 15: Communicating effectively with dementia patients is a challenge to psychiatric nurses.

71. Therapeutic ways to engage the elderly to be stimulated include all of the following EXCEPT:
A. singing to or with the patients C. providing a picture album
B. listening to old familiar music D. playing complex computer games

72. The LEAST area of satisfying communication with the elderly is on:
A. recall of their courtship and love life C. fashion shown in personal pictures
B. psychological trauma D. memories of their teenage parties

50
73. To maximize communication with the elderly, which of the following ways of verbal communication should the nurse
use minimally?
A. asking simple questions C. restating
B. guiding the patient one step at a time D. asking for an open ended response

74. Unconditional positive regard for the elderly is BEST achieved when the nurse-patient relationship is based upon:
A. vocational choices C. addressing economic problems
B. keeping D. providing warmth and caring

75. An optimal supportive environment for a person with dementia includes all, EXCEPT:
A. engaging the elderly in logical and abstract thinking
B. developing a realistic uniformed and consistent daily schedule
C. handling memory loss by distracting or diverting patient’s attention to something more positive
D. simple nurse care for the patient day after day

Situation 16: Harry, age 36 is admitted to the psychiatry unit in an acute manic episode of bipolar disorder.

76. Harry’s condition is a disturbance of:


A. sensorium C. the CNS
B. cognition D. affect

77. In the ward, Harry manifests excess energy and is difficult for him to sit still. The MOST useful activity for him would
be:
A. delivering supply of linen to other rooms C. conducting a drama workshop
B. engaging in activity therapy and group exercises D. painting a mural with other patients

78. Harry’s disruptive behavior on the unit has been increasingly annoying other clients. Which of the following would the
nurse do?
A. set limits on his behavior and be consistent in approach
B. make a rigid structured plan that he will have to follow
C. ignore his behavior
D. tell him that he is annoying other clients and isolate him in his room

79. Harry approaches you and says “I have awarded Top Salesman of the Year. My boss will come and celebarate with
me”. You know for a fact that this is NOT true. Harry is demonstrating a delusion of:
A. achievement C. influence
B. reference D. grandeur

80. The defense mechanism utilized by manic patients to cover up depression is:
A. displacement C. compensation
B. denial D. reaction formation

Situation 17: The nurse observed that Marie, age 28 years old had not been participating in activity therapies.

81. Which of the following remarks from the nursing attendants indicates a need for further teaching and observation?
A. Marie made no response to an invitation to play
B. Marie preferred to sit at the bench and watch the ballgame
C. Marie is aloof and indifferent to co-patients
D. Marie read a book while other patients played a ballgame

82. Marie said “I don’t like to be a part of it. Look, they are being laughed at because they are singing and acting like
children”. The nurse notes that in order for the activity therapy to be therapeutic:
A. the staff should decide solely what activities to be done and what rules apply
B. patients should be allowed solely to decide what they want to do on their own
C. ballgames should be limited to male patients only
D. age and needs of patient should be considered in the choice of games

83. To encourage active participation among patients, it is BEST to plan activities they can engage in through a:
51
A. one to one interaction C. checklist
B. community meeting D. feedback evaluation forum

84. In planning activities for the patients, it is essential to consider FOREMOST:


A. safety and security C. novelty and creativity
B. variety and fun D. excitement and challenge

85. Adults, “ singing and acting like children” is a form of:


A. displacement C. sublimation
B. regression D. compensation

Situation 18: A research was conducted on. “ The effects of the nurse expressive role in the reduction of anxiety in
patients who will undergo nasogastric tube insertion”.

86. The research is experimental which means that utilized:


A. an interview scheduled C. a questionnaire
B. a survey D. control and experimental groups

87. The independent variable is:


A. patients C. reduction of anxiety
B. expressive role D. nasogastric insertion

88. The dependent variable is:


A. nasogastric ingestion C. expressive role
B. patients D. reduction of anxiety

89. This study was done to check the procedures of the research:
A. protest study C. pilot study
B. proposal D. test-retest

90. Which of the following is NOT a characteristic of experimental research?


A. value plays a significant part in research
B. research ideas are capable of being tested
C. results of tests made on the groups are compared
D. research subjects are randomly selected and placed into groups for the purpose of manipulation

Situation 19: The client is walking to and fro along the hospital corridor and swinging her hands. She thinks she is going
crazy. She is having palpitations, rapid pulse and hyperventilation.

91. Your nursing intervention should be to:


A. encourage her to participate in an ongoing activity in the ward
B. stay with her in a calm environment
C. tell her to stop thinking that she is going crazy
D. isolate the client in a quiet room

92. Which of the following is a behavioral manifestation of anxiety?


A. hyperventilation C. panic
B. tachycardia D. rapid pulse

93. Which of the following statements about anxiety is NOT true?


A. anxiety is contagious
B. repressed unresolved conflicts create anxiety
C. anxiety at any level is destructive, hence a threat to a persons well being
D. reactions to a perceived threat maybe real or imaginary

94. There is increased tension and reduced ability to perceive and communicate, hence “selective attention” in this level
of anxiety:
52
A. moderate C. panic
B. severe D. mild

95. This medication is indicated for anxious patients:


A. Tofranil (Imipramine HCL) C. Valium (Diazepam)
B. Haldol (Haloperidol) D. Thorazine (Chlorpromazine)

Situation 20: Andy, 30 years old, was admitted to the Psychiatric Ward because of religious preoccupation, deterioration
in self-care and disturbed thoughts. He believes that he has committed a lot of sins. He is threatened by people reaching
out to him. His fasting for several days was not sufficient for him to feel forgiven.

96. Andy is demonstrating:


A. religious delusion C. somatic delusion
B. delusion of grandeur D. delusion of being controlled

97. A delusion is:


A. psychomotor disturbance C. disturbance of thought
B. mood disturbance D. disturbance of perception

98. The nursing goal for Andy is to:


A. have him see a priest for confession
B. encourage him to pray to atone for his sins
C. help him develop a positive self image
D. socialize him with a group to keep him in touch with reality

99. As Andy talks about his sins that he believes make people look down upon him. It is BEST to:
A. agree with him and sympathize how sinful he has really been
B. explore the nature of his sins
C. explain that he is depreciating himself too much
D. acknowledge how he feels and focus on reality oriented topics

100. The psychosocial task that Andy needs to work on is a sense of:
A. identity C. autonomy
B. trust D. intimacy

----------------------------------------------------------------------------------------------------------------------------------------------------

53
54

Vous aimerez peut-être aussi